*NURSING > EXAM > NRNP 6531 Final Exam compliation (GRADED A) Questions and Answers solutions | 100% Verified EXAM TES (All)

NRNP 6531 Final Exam compliation (GRADED A) Questions and Answers solutions | 100% Verified EXAM TESTBANK.

Document Content and Description Below

User Course NURS-6531N-8,Adv. Practice Care of Adults.2018 Summer Qtr 05/29-08/19-PT27 Test Midterm Exam Status Needs Grading Attempt Score 0 out of 100 points Time Elapsed 1 hour, 53 m... inutes out of 1 hour and 50 minutes Results Displayed All Answers, Submitted Answers, Correct Answers • Question 1 0 out of 0 points When completing this quiz, did you comply with Walden University’s Code of Conduct including the expectations for academic integrity? Selected Answer: Ye s Answers: Ye s No • Question 2 The most common cancer found on the auricle is: Selected Answer: Basal cell carcinoma Answers: Actinic keratosis Basal cell carcinoma Squamous cell carcinoma Acral-lentiginous melanoma • Question 3 1 out of 1 points 0 out of 1 points Which of the following medication classes should be avoided in patients with acute or chronic bronchitis because it will contribute to ventilation-perfusion mismatch in the patient? Selected Answer: Anticholinerg ics Answers: Xanthines Antihistimine s Steroids Anticholinerg ics • Question 4 0 out of 1 points A 47 year old male patient presents to the clinic with a single episode of a moderate amount of bright red rectal bleeding. On examination, external hemorrhoids are noted. How should the nurse practitioner proceed? Selected Answer: Instruct the patient on measures to prevent hemorrhoids such as bowel habits and diet. Answers: Instruct the patient on measures to prevent hemorrhoids such as bowel habits and diet. Order a topical hemorrhoid cream along with a stool softener. Refer the patient for a barium enema and sigmoidoscopy. Refer the patient for a surgical hemorrhoidectomy. • Question 5 0 out of 1 points Which of the following patient characteristics are associated with chronic bronchitis? Selected Answer: Underweight, pink skin, and increased respiratory rate Answers: Overweight, cyanosis, and normal or slightly increased respiratory rate Underweight, pink skin, and increased respiratory rate Overweight, pink skin, and normal or slightly increased respiratory rate Normal weight, cyanosis, and greatly increased respiratory rate • Question 6 1 out of 1 points A 65-year-old female with a past medical history of hypertension, hyperlipidemia, and polymyalgia rheumatica presents to urgent care with new onset left lower quadrant pain. Her current medications include omeprazole 20 milligrams po daily, lisinopril 20 milligrams po daily, simvastatin 20 milligrams po daily, and prednisone 12 milligrams po daily. The nurse practitioner suspects acute diverticulitis and possibly an abscess. The most appropriate diagnostic test for this patient at this time is: Selected Answer: CT scan Answers: CBC/diff Erythrocyte sedimentation rate Abdominal ultrasound CT scan • Question 7 1 out of 1 points A patient reports “something flew in my eye” about an hour ago while he was splitting logs. If there were a foreign body in his eye, the nurse practitioner would expect to find all except: Selected Answer: Purulent drainage Answers: Purulent drainage Tearing Photophobia A positive fluorescein stain • Question 8 1 out of 1 points A 21 year old college student presents to the student health center with copious, markedly purulent discharge from her left eye. The nurse practitioner student should suspect: Selected Answer: Gonococcal conjunctivitis Answers: Viral conjunctivitis Common pink eye Gonococcal conjunctivitis Allergic conjunctivitis • Question 9 0 out of 1 points A 35 year old man presents with radicular pain followed by the appearance of grouped vesicles consisting of about 15 lesions across 3 different thoracic dermatomes. He complains of pain, burning, and itching. The nurse practitioner should suspect: Selected Answer: A complicated case of shingles and prescribe acyclovir, an analgesic, and a topical cortisone cream Answers: A common case of shingles and prescribe an analgesic and an antiviral agent A complicated case of shingles and prescribe acyclovir, an analgesic, and a topical cortisone cream Herpes zoster and consider that this patient may be immunocompromised A recurrence of chickenpox and treat the patient’s symptoms • Question 10 0 out of 1 points Which type of lung cancer has the poorest prognosis? Selected Answer: Adenocarcinoma Answers: Adenocarcinoma Epidermoid carcinoma Small cell carcinoma Large cell carcinoma • Question 11 1 out of 1 points An 83-year-old female presents to the office complaining of diarrhea for several days. She explains she has even had fecal incontinence one time. She describes loose stools 3– 4 times a day for several weeks and denies fever, chills, pain, recent antibiotic use. The history suggests that the patient has: Selected Answer: Chronic diarrhea Answers: Acute diarrhea Chronic diarrhea Irritable bowel Functional bowel disease • Question 12 1 out of 1 points Margaret, age 32, comes into the office with painful joints and a distinctive rash in a butterfly distribution on her face. The rash has red papules and plaques with a fine scale. What do you suspect? Selected Answer: Systemic lupus erythematosus Answers: An allergic reaction Relapsing polychondritis Lymphocytoma cutis Systemic lupus erythematosus • Question 13 1 out of 1 points Antibiotic administration has been demonstrated to be of little benefit to the treatment of which of the following disease processes? Selected Answer: Acute bronchitis Answers: Chronic sinusitis Acute bronchitis Bacterial pneumonia Acute exacerbation of chronic bronchitis • Question 14 0 out of 1 points Lisa, age 49, has daily symptoms of asthma. She uses her inhaled short-acting beta-2 agonist daily. Her exacerbations affect her activities and they occur at least twice weekly and may last for days. She is affected more than once weekly during the night with an exacerbation. Which category of asthma severity is Lisa in? Selected Answer: Mild persistent Answers: Mild intermittent Mild persistent Moderate persistent • Question 15 0 out of 1 points Which of the following is the most appropriate therapeutic regimen for an adult patient with no known allergies diagnosed with group A B-hemolytic strep? Selected Answer: None of the above Answers: Penicillin V 500 milligrams PO every 8 hours for 10 days Ampicillin 250 milligrams PO twice a day for 10 days Clarithromycin 500 milligrams po daily for 7 days None of the above • Question 16 0 out of 1 points A cashier complains of dull ache and pressure sensation in her lower legs. It is relieved by leg elevation. She occasionally has edema in her lower legs at the end of the day. What is the most likely cause of these problems? Selected Answer: Arterial insufficiency Answers: Congestive heart failure Varicose veins Deep vein thrombosis Arterial insufficiency • Question 17 Which statement below is correct about pertussis? Selected 1 out of 1 points Answer: It is also called whooping cough Answers: It is also called whooping cough It begins with symptoms like strep throat It lasts about 3 weeks It occurs most commonly in toddlers and young children • Question 18 0 out of 1 points Which of the following is the most important diagnosis to rule out in the adult patient with acute bronchitis? Selected Answer: Asthma Answers: Pneumo nia Asthma Sinusitis Pertussis • Question 19 1 out of 1 points A 70 year old patient presents with left lower quadrant (LLQ) abdominal pain, a markedly tender palpable abdominal wall, fever, and leukocytosis. Of the following terms, which correctly describes the suspected condition? Selected Answer: Diverticulit is Answers: Diverticulo sis Diverticula Diverticulit is Diverticulu m • Question 20 1 out of 1 points Sylvia, age 83, presents with a 3 day history of pain and burning in the left forehead. This morning she noticed a rash with erythematous papules in that site. What do you suspect? Selected Answer: Herpes zoster Answers: Varicella Herpes zoster Syphilis Rubella • Question 21 0 out of 1 points A 33-year-old female is admitted with acute pancreatitis. The nurse practitioner knows that the most common cause of pancreatitis is: Selected Answer: Medicatio ns Answers: Alcohol Gallstone s Medicatio ns Pregnanc y • Question 22 1 out of 1 points When a patient presents with symptoms of acute gallbladder disease, what is the appropriate nurse practitioner action? Selected Answer: Order an abdominal ultrasound Answers: Order abdominal x-rays Order an abdominal ultrasound Refer the patient to a surgeon for evaluation Prescribe pain medication • Question 23 A false-positive result with the fecal occult blood test can result from: Selected Answer: stool that has been stored before testing Answers: ingestion of large amounts of vitamin C a high dietary intake of rare cooked beef a colonic neoplasm that is not bleeding stool that has been stored before testing • Question 24 0 out of 1 points 0 out of 1 points A 76-year-old male complains of weight loss, nausea, vomiting, abdominal cramping and pain. Physical findings include an abdominal mass and stool positive for occult blood. The nurse practitioner pain suspects a tumor in the small intestine. The best diagnostic test for this patient is: Selected Answer: Colonoscopy Answers: Colonoscopy Small bowel follow- through Barium enema CT abdomen • Question 25 1 out of 1 points A patient presents to urgent care complaining of dyspnea, fatigue, and lower extremity edema. The echocardiogram reveals and ejection fraction of 38%. The nurse practitioner knows that these findings are consistent with: Selected Answer: Systolic heart failure Answers: Mitral regurgitation Systolic heart failure Cardiac myxoma Diastolic heart failure • Question 26 1 out of 1 points Maxine, Age 76, has just been given a diagnosis of pneumonia. Which of the following is an indication that she should be hospitalized? Selected Answer: Multilobar involvement on chest x-ray with the inability to take oral medications Answers: Multilobar involvement on chest x-ray with the inability to take oral medications Alert and oriented, slightly high but stable vital signs, and no one to take care of her at home Sputum and gram positive organisms A complete blood count showing leukocytosis • Question 27 0 out of 1 points A 55 year old man is diagnosed with basal cell carcinoma. The nurse practitioner correctly tells him: Selected Answer: “It is the most common cause of death in patients with skin cancer.” Answers: “It is the most common cause of death in patients with skin cancer.” “It can be cured with surgical excision or radiation therapy.” “It is a slow growing skin cancer that rarely undergoes malignant changes.” “It can be cured using 5-flurouracil cream twice daily for 2 to 4 weeks.” • Question 28 1 out of 1 points Expected spirometry readings when the patient has chronic emphysema include: Selected Answer: Increased total lung capacity (TLC) Answers: Decreased residual volume (RV) Increased vital capacity (VC) Increased forced expiratory volume (FEV-1) Increased total lung capacity (TLC) • Question 29 1 out of 1 points An 80-year-old male admits to difficulty swallowing during the review of systems. The nurse practitioner recognizes the differential diagnosis for this patient’s dysphagia is: Selected Answer: A and C Answers: Esophageal cancer Chest pain GERD A and C All of the above • Question 30 0 out of 1 points A 40 year old female with history of frequent sun exposure presents with a multicolored lesion on her back. It has irregular borders and is about 11mm in diameter. What should the nurse practitioner suspect? Selected Answer: Basal cell carcinoma Answers: Squamous cell carcinoma Malignant melanoma A common nevus Basal cell carcinoma • Question 31 1 out of 1 points Which of the following is not a goal of treatment for the patient with cystic fibrosis? Selected Answer: Replace water-soluble vitamins Answers: Prevent intestinal obstruction Provide adequate nutrition Promote clearance of secretions Replace water-soluble vitamins • Question 32 1 out of 1 points The nurse practitioner is performing a physical exam on a middle-aged African-American man. Which of the following areas is a common site for melanomas in African-Americans and other dark-skinned individuals? Selected Answer: B and C Answers: Scalp Nails Feet B and C All of the above • Question 33 1 out of 1 points An adult presents with tinea corporis. Which item below is a risk factor for its development? Selected Answer: Topical steroid use Answers: Topical steroid use Topical antibiotic use A recent laceration Cold climates • Question 34 0 out of 1 points A patient has experienced nausea and vomiting, headache, malaise, low grade fever, abdominal cramps, and watery diarrhea for 72 hours. His white count is elevated with a shift to the left. He is requesting medication for diarrhea. What is the most appropriate response? Selected Answer: Order stool cultures. Answers: Prescribe loperamide (Immodium) or atropine-diphenoxylate (Lomotil) and a clear liquid diet for 24 hours. Prescribe a broad-spectrum antibiotic such as ciprofloxacin (Cipro), and symptom management. Offer an anti-emetic medication such as ondansetron (Zofran) and provide oral fluid and electrolyte replacement instruction. Order stool cultures. • Question 35 1 out of 1 points Janine, age 29, has numerous transient lesions that come and go, and she is diagnosed with urticaria. What do you order? Selected Answer: Antihistami nes Answers: Aspirin NSAIDs Opioids Antihistami nes • Question 36 0 out of 1 points Of the following signs and symptoms of congestive heart failure (CHF), the earliest clinical manifestation is: Selected Answer: Peripheral edema Answers: Peripheral edema Weight gain Shortness of breath Nocturnal dyspnea • Question 37 1 out of 1 points A 16 year old male presents with mild sore throat, fever, fatigue, posterior cervical adenopathy, and palatine petechiae. Without a definitive diagnosis for this patient, what drug would be least appropriate to prescribe? Selected Answer: Amoxicillin Answers: Ibuprofen Erythromyci n Amoxicillin Acetaminop hen • Question 38 1 out of 1 points A 70 year old man who walks 2 miles every day complains of pain in his left calf when he is walking. The problem has gotten gradually worse and now he is unable to complete his 2 mile walk. What question asked during the history, if answered affirmatively, would suggest a diagnosis of arteriosclerosis obliterans? Selected Answer: “Is your leg pain relieved by rest?” Answers: “Are you wearing your usual shoes?” “Do you also have chest pain when you have leg pain?” “Is your leg pain relieved by rest?” “Do you ever have the same pain in the other leg?” • Question 39 Which of the following statements about malignant melanomas is true? Selected Answer: They usually occur in older adult males Answers: They usually occur in older adult males The patient has no family history of melanoma They are common in blacks The prognosis is directly related to the thickness of the lesion • Question 40 0 out of 1 points 1 out of 1 points Sheila, age 78, presents with a chief complaint of waking up during the night coughing. You examine her and find an S3 heart sound, pulmonary crackles that do not clear with coughing, and peripheral edema. What do you suspect? Selected Answer: Heart failure Answers: Asthma Nocturnal allergies Valvular disease Heart failure • Question 41 0 out of 1 points Which antibiotic would be the most effective in treating community acquired pneumonia (CAP) in a young adult without any comorbid conditions? Selected Answer: Doxycycline (Vibramycin) Answers: Erythromycin Clarithromycin (Biaxin) Doxycycline (Vibramycin) Penicillin • Question 42 1 out of 1 points Which of the following dermatologic vehicles are the most effective in absorbing moisture and decreasing friction? Selected Answer: Powde rs Answers: Powde rs Gels Cream s Lotion • Question 43 1 out of 1 points A 70 year old patient presents with a slightly raised, scaly, erythematous patch on her forehead. She admits to having been a “sun worshiper.” The nurse practitioner suspects actinic keratosis. This lesion is a precursor to: Selected Answer: Squamous cell carcinoma Answers: Squamous cell carcinoma Basal cell carcinoma Malignant melanoma Acne vulgaris • Question 44 1 out of 1 points An elderly patient is being seen in the clinic for complaint of “weak spells” relieved by sitting or lying down. How should the nurse practitioner proceed with the physical examination? Selected Answer: Compare the patient’s blood pressure lying first, then sitting, and then standing. Answers: Assist the patient to a standing position and take her blood pressure. Assess the patient’s cranial nerves. Compare the patient’s blood pressure lying first, then sitting, and then standing. Compare the amplitude of the patient’s radial and pedal pulses. • Question 45 1 out of 1 points What oral medication might be used to treat chronic cholethiasis in a patient who is a poor candidate for surgery? Selected Answer: Ursodiol Answers: Ursodiol Ibuprofen Prednisone Surgery is the only answer • Question 46 0 out of 1 points A 46-year-old female with a past medical history of diabetes presents with a swollen, erythematous right auricle and is diagnosed with malignant otitis externa. The nurse practitioner knows that the most likely causative organism for this patient’s problem is: Selected Answer: Staphylococcus aureus Answers: Staphylococcus aureus Group A beta hemolytic streptococcus Haemophilus influenza Pseudomonas aeruginosa • Question 47 Which of the following is not a symptom of irritable bowel syndrome? Selected Answer: Painful diarrhea Answers: Painful diarrhea Painful constipation Cramping and abdominal pain 0 out of 1 points Weight loss • Question 48 1 out of 1 points A patient comes in complaining of 1 week of pain in the posterior neck with difficulty turning the head to the right. What additional history is needed? Selected Answer: Any recent trauma Answers: Any recent trauma Difficulty swallowing Stiffness in the right shoulder Change in sleeping habits • Question 49 0 out of 1 points Marvin, age 56, is a smoker with diabetes. He has just been diagnosed as hypertensive. Which of the following drugs has the potential to cause the development of bronchial asthma and inhibit gluconeogenesis? Selected Answer: ACE Inhibitor Answers: ACE Inhibitor Beta Blocker Calcium channel blocker Diuretic • Question 50 1 out of 1 points The differential diagnosis for a patient complaining of a sore throat includes which of the following? Selected Answer: A, B, and C Answers: Gonococcal infection Thrush Leukoplakia B only A, B, and C • Question 51 0 out of 1 points A patient presents to the primary care provider complaining of a rash on his right forehead that started yesterday and is burning and painful. The physical exam reveals an erythematous, maculopapular rash that extends over the patient’s right eye to his upper right forehead. Based on the history and examination, the most likely cause of this patient’s symptoms is: Selected Answer: Rhus dermatitis Answers: Rhus dermatitis Ophthalmic zoster Chemosis Optic neuritis • Question 52 0 out of 1 points Before initiating an HMG CoA-reductase inhibitor for hyperlipidemia, the nurse practitioner orders liver function studies. The patient’s aminotransferase (ALT) is elevated. What laboratory test(s) should be ordered? Selected Answer: Serum cholesterol with HDL and LDL Answers: Serologic markers for hepatitis Serum bilirubin Serum cholesterol with HDL and LDL A liver biopsy • Question 53 1 out of 1 points A patient with elevated lipids has been started on lovastatin. After 3 weeks of therapy, he calls to report generalized muscle aches. The nurse practitioner should suspect: Selected Answer: Rhabdomyolysis Answers: A drug interaction Hepatic dysfunction Hypersensitivity to lovastatin Rhabdomyolysis • Question 54 Treatment of acute vertigo includes: Selected Answer: Bedrest and an antihistamine 1 out of 1 points Answers: Bedrest and an antihistamine Fluids and a decongestant A sedative and decongestant Rest and a low sodium diet • Question 55 Treatment of H.pylori includes which of the following? Selected Answer: A, B, and C Answers: Proton pump inhibitor Antibiotic therapy Bismuth subsalicylate A and B A, B, and C • Question 56 1 out of 1 points 1 out of 1 points Carl, age 78, is brought to the office by his son, who states that his father has been unable to see clearly since last night. Carl reports that his vision is “like looking through a veil.” He also sees floaters and flashing lights but is not having any pain. What do you suspect? Selected Answer: Retinal detachment Answers: Cataracts Glaucoma Retinal detachment Iritis • Question 57 In order to decrease deaths from lung cancer: 1 out of 1 points Selected Answer: Patients should be counseled to quit smoking Answers: All smokers should be screened annually All patients should be screened annually Only high risk patients should be screened routinely Patients should be counseled to quit smoking • Question 58 1 out of 1 points John, age 33, has a total cholesterol level of 188 mg/dL. How often should he be screened for hypercholesterolemia? Selected Answer: Every 5 years Answers: Every 5 years Every 2 years Every year Whenever blood work is done • Question 59 1 out of 1 points Mort is hypertensive. Which of the following factors influenced your choice of using an alpha blocker as the antihypertensive medication? Selected Answer: Mort has benign prostatic hyperplasia Answers: Mort is black Mort also has congestive heart failure Mort has benign prostatic hyperplasia Mort has frequent migraine headaches • Question 60 1 out of 1 points John, age 59, presents with recurrent, sharply circumscribed red papules and plaques with a powdery white scale on the extensor aspect of his elbows and knees. What do you suspect? Selected Answer: Psoriasis Answers: Actinic keratosis Eczema Psoriasis Seborrheic dermatitis • Question 61 1 out of 1 points Harriet, a 79-year-old woman, comes to your office every 3 months for follow up on her hypertension. Her medications include one baby aspirin daily, Lisinopril 5mg daily, and Calcium 1500 mg daily. At today’s visit. Her blood pressure is 170/89. According to JNC VIII guidelines, what should you do next to control Harriet’s blood pressure? Selected Answer: Add a thiazide diuretic to the Lisinopril 5mg daily Answers: Increase her Lisinopril to 20mg daily Add a thiazide diuretic to the Lisinopril 5mg daily Discontinue the Lisinopril and start a combination of ACE Inhibitor and calcium channel blocker Discontinue the Lisinopril and start a diuretic • Question 62 0 out of 1 points An active 65-year-old man under your care has known acquired valvular aortic stenosis and mitral regurgitation. He also has a history of infectious endocarditis. He has recently been told he needs elective replacement of his aortic valve. When he comes into the office you discover that he has 10 remaining teeth in poor repair. Your recommendation would be to: Selected Answer: coordinate with his cardiac and oral surgeons to have the tooth extractions and valve replacement done at the same time to reduce the risk of anesthetic complications. Answers: defer any further dental work until his valve replacement is completed instruct him to have dental extraction done cautiously, having no more than 2 teeth per visit removed. suggest he consult with his oral surgeon about having all the teeth removed at once and receiving appropriate antibiotic prophylaxis coordinate with his cardiac and oral surgeons to have the tooth extractions and valve replacement done at the same time to reduce the risk of anesthetic complications. • Question 63 Appropriate therapy for peptic ulcer disease (PUD) is: Selected Answer: Dependent on cessation of NSAID use Answers: Primarily by eradication of infection Based on etiology Aimed at diminishing prostaglandin synthesis Dependent on cessation of NSAID use • Question 64 0 out of 1 points 1 out of 1 points Shirley, age 58, has been a diabetic for 7 years. Her blood pressure is normal. Other than her diabetes medications, what would you prescribe today during her routine office visit? Selected Answer: An ACE Inhibitor Answers: A calcium channel blocker A beta blocker An ACE Inhibitor No hypertension medication • Question 65 1 out of 1 points Medicare is a federal program administered by the Centers for Medicare and Medicaid Services (CMS). The CMS has developed guidelines for Evaluation and Management coding, which all providers are expected to follow when coding patient visits for reimbursement. Which of the following is an important consideration regarding billing practices? Selected Answer: Failing to bill for billable services will lead to unnecessarily low revenues Answers: It is important to “undercode” so that one does not get charged with Medicare fraud The practice of “overcoding” is essential in this age of decreasing reimbursements Failing to bill for billable services will lead to unnecessarily low revenues Time spent with the patient is a very important determinant of billing • Question 66 1 out of 1 points A 2 year old presents with a white pupillary reflex. What is the most likely cause of this finding? Selected Answer: Retinoblastom a Answers: Viral conjunctivitis Glaucoma Corneal abrasion Retinoblastom a • Question 67 0 out of 1 points Harvey has had Meniere’s disease for several years. He has some hearing loss but now has persistent vertigo. What treatment might be instituted to relieve the vertigo? Selected Answer: A labyrinthectomy Answers: Pharmacological therapy A labyrinthectomy A vestibular neurectomy Wearing an earplug in the ear with the most hearing loss • Question 68 Which of the following is not a risk factor for coronary arterial insufficiency? Selected Answer: Hyperhomocystein emia Answers: Hyperhomocystein emia Smoking Genetic factors Alcohol ingestion • Question 69 0 out of 1 points 1 out of 1 points An 18-year-old female presents to the urgent care center complaining of severe pruritus in both eyes that started 2 days ago. Associated symptoms include a headache and fatigue. On examination, the nurse practitioner notes some clear discharge from both eyes and some erythema of the eyelids and surrounding skin. Which of the following is the most likely cause of this patient’s symptoms? Selected Answer: Allergic conjunctivitis Answers: Allergic conjunctivitis Bacterial conjunctivitis Gonococcal conjunctivitis Viral conjunctivitis • Question 70 0 out of 1 points A 20 year old is diagnosed with mild persistent asthma. What drug combination would be most effective in keeping him symptom-free? Selected Answer: A long-acting bronchodilator Answers: A long-acting bronchodilator An inhaled corticosteroid and cromolyn Theophylline and a short acting bronchodilator A bronchodilator PRN and an inhaled corticosteroid • Question 71 0 out of 1 points Acute rheumatic fever is an inflammatory disease which can follow infection with: Selected Answer: Β-hemolytic Streptococcus Answers: Group A Streptococcus Staphlococcus areus Β-hemolytic Streptococcus Streptococcus pyogenes • Question 72 0 out of 1 points A 60 year old male diabetic patient presents with redness, tenderness, and edema of the left lateral aspect of his face. His left eyelid is grossly edematous. He reports history of a toothache in the past week which “is better.” His temperature is 100°F and pulse is 102 bpm. The most appropriate initial action is to: Selected Answer: Start an oral antibiotic, mouth swishes with an oral anti-infective, and an analgesic Answers: Start an oral antibiotic, refer the patient to a dentist immediately, and follow up within 3 days Order mandibular x-rays and question the patient about physical abuse Start an oral antibiotic, mouth swishes with an oral anti-infective, and an analgesic Initiate a parenteral antibiotic and consider hospital admission • Question 73 1 out of 1 points If a patient presents with a deep aching, red eye and there is no discharge, you should suspect: Selected Answer: Iritis Answers: Iritis Allergic conjunctivitis Viral conjunctivitis Bacterial conjunctivitis • Question 74 1 out of 1 points The National Cholesterol Education Program’s Adult Treatment Panel III recommends that the goal for low density lipoproteins in high risk patients be less than: Selected Answer: 100 mg/dL Answers: 160 mg/dL 130 mg/dL 100 mg/dL 70 mg/dL • Question 75 1 out of 1 points A patient presents with classic symptoms of gastroesophageal reflux disease (GERD). He is instructed on life style modifications and drug therapy for 8 weeks. Three months later he returns, reporting that he was “fine” as long as he took the medication. The most appropriate next step is: Selected Answer: Investigation with endoscopy, manometry, and/or pH testing Answers: Referral for surgical intervention such as a partial or complete fundoplication Dependent upon how sever the practitioner believes the condition To repeat the 8 week course of drug therapy while continuing lifestyle modifications Investigation with endoscopy, manometry, and/or pH testing • Question 76 0 out of 1 points Group A β-hemolytic streptococcal (GABHS) pharyngitis is most common in which age group? Selected Answer: Under 3 years of age Answers: Under 3 years of age Preschool children 6 to 12 years of age Adolescents • Question 77 The most appropriate treatment for a child with mild croup is: Selected Answer: A decongestant Answers: A bronchodilator An antibiotic A decongestant A cool mist vaporizer • Question 78 0 out of 1 points 0 out of 1 points A child complains that his “throat hurts” with swallowing. His voice is very “throaty” and he is hyperextending his neck to talk. Examination reveals asymmetrical swelling of his tonsils. His uvula is deviated to the left. What is the most likely diagnosis? Selected Answer: Epiglottitis Answers: Peritonsillar abscess Thyroiditis Mononucleosis Epiglottitis • Question 79 1 out of 1 points Salmeterol (Servent) is prescribed for a patient with asthma. What is the most important teaching point about this medication? Selected Answer: It is not effective during an acute asthma attack. Answers: It is not effective during an acute asthma attack. It may take 2 to 3 days to begin working. This drug works within 10 minutes. This drug may be used by patients 6 years and older. • Question 80 0 out of 1 points Which intervention listed below is safe for long term use by an adult with constipation? Selected Answer: Stool softeners Answers: Bulk-forming agents Stool softeners Laxatives Osmotic agents • Question 81 1 out of 1 points A 40 year old presents with a hordeolum. The nurse practitioner teaches the patient to: Selected Answer: Apply a topical antibiotic and warm compresses. Answers: Apply a topical antibiotic and warm compresses. Apply cool compresses and avoid touching the hordeolum. Use an oral antibiotic and eye flushes. Apply light palpation to facilitate drainage. • Question 82 1 out of 1 points Sarah has allergic rhinitis and is currently being bothered by nasal congestion. Which of the following meds ordered for allergic rhinitis would be most appropriate? Selected Answer: A decongestant nasal spray Answers: An antihistamine intranasal spray A decongestant nasal spray Ipratropium Omalizumab • Question 83 What is the Gold standard for the diagnosis of asthma? Selected Answer: Spirometry Answers: Patient’s perception of clogged airways Validated quality-of-life questionnaires Bronchoscopy Spirometry • Question 84 1 out of 1 points 0 out of 1 points A patient complains of “an aggravating cough for the past 6 weeks.” There is no physiological cause for the cough. Which medication is most likely causing the cough? Selected Answer: Methyldopa Answers: Methyldopa Enalapril Amlodipine Hydrochlorothia zide • Question 85 1 out of 1 points Stacy, age 27, states that she has painless, white, slightly raised patches in her mouth. They are probably caused by: Selected Answer: Candidiasis Answers: Herpes simplex Aphthous ulcers Candidiasis Oral cancer • Question 86 Risk factors for acute otitis media (AOM) include all of the following except: Selected Answer: African-American ethnicity Answers: Household cigarette smoke Group daycare attendance Sibling history of acute otitis media African-American ethnicity • Question 87 1 out of 1 points 1 out of 1 points Which of the following can result from chronic inflammation of a meibomian gland? Selected Answer: A chalazion Answers: A chalazion Uveitis Keratitis A pterygiu m • Question 88 0 out of 1 points What conditions must be met for you to bill “incident to” the physician, receiving 100% reimbursement from Medicare? Selected Answer: You must initiate the plan of care for the patient Answers: You must initiate the plan of care for the patient The physician must be on-site and engaged in patient care You must be employed as an independent contractor You must be the main health care provider who sees the patient • Question 89 Of the following choices, the least likely cause of cough is: Selected Answer: Acute pharyngitis Answers: Asthma Gastroesophageal reflux Acute pharyngitis Allergic rhinitis • Question 90 1 out of 1 points 1 out of 1 points The most common correlate(s) with chronic bronchitis and emphysema is(are): Selected Answer: Cigarette smoking Answers: Familial and genetic factors Cigarette smoking Air pollution Occupational environment • Question 91 0 out of 1 points Which choice below is least effective for alleviating symptoms of the common cold? Selected Answer: Topical decongestants Answers: Antihistamines Oral decongestants Topical decongestants Antipyretics • Question 92 0 out of 1 points When teaching a patient with hypertension about restricting sodium, you would include which of the following instructions? Selected Answer: Diets with markedly reduced intakes of sodium may be associated with other beneficial effects beyond blood pressure control Answers: Diets with markedly reduced intakes of sodium may be associated with other beneficial effects beyond blood pressure control Sodium restriction can cause serious adverse effects A goal of 3 g of sodium chloride or 1.2 g of sodium per day is easily achievable Seventy-five of sodium intake is derived from processed foods • Question 93 Which of the following heart murmurs warrants the greatest concern? 1 out of 1 points Selected Answer: Diastolic murmur Answers: Systolic murmur Venous hum murmur Diastolic murmur Flow murmur • Question 94 1 out of 1 points A patient presents with an inflamed upper eyelid margin. The conjunctiva is red and there is particulate matter along the upper eyelid. The patient complains of a sensation that “there is something in my eye.” What is the diagnosis and how should it be treated? Selected Answer: Blepharitis; treat with warm compresses and gentle debridement with a cotton swab Answers: Hordeolum; treat with a topical antibiotic and warm compress Conjunctivitis; treat with topical antibiotic and warm compresses Blepharitis; treat with warm compresses and gentle debridement with a cotton swab Chalazion; refer to an ophthalmologist for incision and drainage • Question 95 1 out of 1 points A 57-year-old male presents to urgent care complaining of substernal chest discomfort for the past hour. The EKG reveals ST elevations in Leads II, III, and AVF. The nurse practitioner is aware that these changes are consistent with which myocardial infarction territory? Selected Answer: Inferior wall Answers: Inferior wall Anterior wall Apical wall Lateral wall • Question 96 0 out of 1 points The nurse practitioner observes a tympanic membrane that is opaque, has decreased mobility, and is without bulging or inflammation. The least likely diagnosis for this patient is: Selected Answer: Otitis media with effusion Answers: Acute otitis media (AOM) Otitis media with effusion Mucoid otitis media Serous otitis media • Question 97 0 out of 1 points Alan, age 54, notices a bulge in his midline every time he rises from bed in the morning. You tell him it is a ventral hernia, also known as: Selected Answer: incisional hernia Answers: inguinal hernia epigastric hernia umbilical hernia incisional hernia • Question 98 1 out of 1 points A 58-year-old man is diagnosed with Barrett’s esophagus after an endoscopy. He has no known allergies. Which of the following medications is MOST appropriate to treat this patient’s disorder? Selected Answer: Omeprazole Answers: Omeprazole Ranitidine An antacid None of the above • Question 99 1 out of 1 points Larry, age 66, is a smoker with hyperlipidemia and hypertension. He is 6 months post-MI. To prevent reinfarction, the most important behavior change that he can make is to: Selected Answer: Quit smoking Answers: Quit smoking Maintain aggressive hypertension therapy Stick to a low-fat, low-sodium diet Continue with his exercise program • Question 100 Risk factors for acute arterial insufficiency include which of the following? Selected Answer: All of the above Answers: Recent myocardial infarction Atrial fibrillation Atherosclerosis All of the above • Question 101 Impetigo and folliculitis are usually successfully treated with: Selected Answer: Topical antibiotics Answers: Systemic antibiotics Topical antibiotics Topical steroid creams 1 out of 1 points 1 out of 1 points Cleansing and debridement Quizlet finals What are signs & symptoms of SIADH (Syndrome of inappropriate antidiuretic hormone)? Increased production of ADH (antidiuretic hormone), hyponatremia, concentrated urine (from excess water resorption), elevated urine osmolality, mental status changes from cerebral edema. Diabetes insipidus is associated with what sodium level? Hypernatremia Psychogenic polydipsia results in urine that is: diluted with low osmolality and hyponatremia How would you determine the cause of a patient's AKI who presents with decreased urine output, history of neurogenic bladder, chronic foley, dark urine, and Cr increase from 1.3 to 2.1 over 3 months? Flush the foley catheter to see if urine comes out and assess the patency of the catheter. This action will unblock clogged sediment or biofilm from chronic bacteriuria. When a female patient presents to the ER after sexual assault, what medications should be offered prior to discharge? Ceftriaxone, azithromycin, Plan B, and Metronidazole. Manifestations of Conn syndrome (hyperaldosteronism)? hypernatremia, hypokalemia, and hypertension What causes Cushing syndrome? Increased levels of glucocorticoids, can be exogenous (from therapy) or endogenous (from adenoma or neoplasm). Manifestations of Cushing syndrome? hypertension, truncal obesity, osteoporosis, skin fragility, and hyperglycemia. What differentiates primary adrenocortical insufficiency from secondary adrenocortical insufficiency? Skin hyperpigmentation is present in primary adrenocortical insufficiency What is Trousseau's sign? A carpal spasm elicited by compression of the upper arm with a BP cuff that indicates hypocalcemia. What is Chovstek's sign? A hemifacial tic that is induced by tapping the facial nerve below the maxilla that indicates hypocalcemia. What is Babinski's sign? An upward response (extension) of the hallux when the sole of the foot is stimulated with a blunt instrument. Can identify spinal cord disease in adults. What is Romberg's sign? Loss of balance in standing when eyes are closed. Usually indicating a loss of proprioception or lesion in the cerebellum. What is Homan's sign? pain on passive dorsiflexion of ankle, associated with DVT. What is the clinical presentation of Goodpasture's syndrome? Urinalysis: Specific gravity: 1.020. pH 5.5, 1+ albumin and large blood present. Chest XR positive for bilateral diffuse infiltrates. BUN 30, Cr 3.0 Symptoms: dyspnea with hemoptysis What is Goodpasture syndrome? Damage to alveolar and renal glomerular basement membranes by cytotoxic antibody. Initial treatment for Goodpasture's syndrome? Hospitalization, pulse dose of steroids, begin plasmapheresis and cyclophosphamide therapy. Characteristics of Grave's disease Ophthalmopathy (lid retraction, scleral show, proptosis) and hyperthyroidism What organism causes the formation of a staghorn calculus? Proteus mirabilus What organism causes Toxic shock syndrome? Staphylococcus aureus What electrolyte disturbance is most likely to lead to tetany and neuromuscular irritability? Hypocalcemia Features of hypercalcemia include? "Stones, groans, moans, and bones." Delerium and renal stones When you see hypochloremia, the patient may have? Metabolic Alkalosis Hyperkalemia is associated with what cardiac abnormalities? peaked T-waves, wide QRS, and ventricular arrhythmias. A patient presents with arcus cornea, LDL 285, TG 110, HDL 45, and father died of an MI at age 45. What is his most likely diagnosis? Heterozygous familial hypercholesterolemia A fasting blood glucose level of 130 mg/dL indicates: Diabetes What are risk factors for ectopic pregnancy? smoking, previous tubal surgery, previous ectopic pregnancy, exposure to diethylibestrol, current IUD, PID, advanced maternal age, infertilitiy for more than 2 years. What hormones are most critical to replace in a patient at risk for anterior pituitary insufficiency? Glucocorticoids, and thyroid hormone What is diabetes insipidus? a disorder caused by inadequate amounts of ADH which causes excessive water loss CKD Stage 1 GFR >90 with evidence of renal damage, as indicated by proteinuria. CKD Stage 2 GFR 60-89 CKD Stage 3a GFR 45-59 Upgrade to remove ads Only $2.99/month CKD Stage 3b GFR 30-44 What is the diagnosis and initial treatment of a patient with history of nonfunctional pituitary macroadenoma with severe retro-orbital headache, nausea, vomiting, mental status change, right third nerve palsy with stiff neck, and pituitary adenoma (enlarged) hemorrhage? Pituitary apoplexy dexamethasone 2mg IV q 6hr then surgery if mental status change present. What causes hypocalcemia in patients with end stage renal disease? Decreased conversion of Vitamin D. What condition is associated with recurrent nephrolithiasis? Primary hyperparathyroidism What stabilizes the cardiac membrane in treatment of hyperkalemia? Calcium gluconate What medication acts rapidly to antagonize the membrane effects of potassium in hyperkalemia? Calcium is cardioprotective What is the most common cause of euvolemic hyponatremia? SIADH (syndrome of inappropriate antidiuretic hormone secretion) What causes are associated with hypervolemic hyponatremia? Cirrhosis and CHF What is a long-term complication of radio-active iodine therapy? Hypothyroidism 80% of acute pyelonephritis cases in women are caused by what pathogen? E coli Most common cause of acute scrotal pain in adults? epididymitis What are predisposing factors for hypocalcemia? hypoparathyroidism, pancreatitis, vitamin D deficiency, chronic alcohol use disorder, and medications such as phenytoin, cisplatin, and estrogen. What are prerenal causes of AKI? decrease in extracellular fluid volume, decreased renal blood flow, or altered intrarenal hemodynamics. Post-renal causes of AKI benign prostatic hyperplasia, bladder cancer, nephrolithiasis, neuromuscular disorders, prostate cancer, strictures, trauma What is are intrinsic causes of renal failure? Acute glomerulonephritis, ischemic injury, nephrotoxins, malignant hypertension What classification of drugs cause urinary retention? anticholinergics (ipratropium) Common secondary causes of nephrotic syndrome lupus and diabetes Presenting symptoms of nephrotic syndrome loss of appetite, fatigue, ascites, lower extremity edema, proteinuria, low blood albumin, and hyperlipidemia. What is the initial insulin loading dose for a patient in DKA? 0.1 to 0.15 units/kg/hr with hourly glucose monitoring What is Hashimoto's thyroiditis? autoimmune destruction of thyroid gland, Anti-thyroid stimulating hormone (TSH) receptor antibodies inhibit release of thyroid hormone. Upgrade to remove ads Only $2.99/month What would you do first for an incidental finding of an adrenal mass? Overnight dexamethasone suppression test. Clinical signs of PCOS oligomenorrhea, infertility, hirsutism, obesity, and 2/3 Rotterdam criteria: oligo-ovulation or anovulation clinic of biochemical signs of hyperandrogenism polycystic ovaries on US Treatment for simple cystitis 5 days of nitrofurantoin Define the acronym MUDPILES Acronym for identifying a high anion gap metabolic acidosis: Methanol Uremia Diabetic ketoacidosis Paraldehyde Iron, isoniazid Lactate Ethanol, ethylene glycol Salicylates First-line evaluation of vulvovaginal candidiasis wet mount Prolonged nausea and vomiting can result in: Metabolic alkalosis loss of gastric acid Clinical signs/symptoms of uremic pericarditis worsening anemia, absence of diffuse ST-segment T-wave elevations, fever, pleuritic chest pain, pain that increases in recumbent position. Hemodialysis treatment should be heparin free. What is hepatorenal syndrome? chronic or acute renal disease with advanced hepatic failure and portal hypertension. What complaint is common with patients who have acute on chronic urinary retention? urinary incontinence Aside from potassium chloride as a supplement, what can be used to treat hypokalemia? Magnesium For a patient who is lethargic and hyperkalemic, what is the first line therapy? Calcium gluconate or calcium chloride Priapism is a known side effect of what medication? Trazodone What is the maximum compensation of PaCO2 in metabolic acidosis? 26 signs of preeclampsia BP > 140/90 proteinuria >300 mg of protein in urine in 24 hours headache visual changes 2+ pitting edema Question 2. Which of the following is a potential acquired cause of thrombophilia Homocysteinuria Protein C deficiency Factor V Leiden Antiphospholipid antibodies Question 3 Phalen's test, 90°wrist flexion for 60 seconds, reproduces symptoms of: Ulnar tunnel syndrome Carpal tunnel syndrome Tarsal tunnel syndrome Myofascial pain syndrome Question 4 Which patient would benefit most from screening for type 2 diabetes? A 30 year old female with unintended weight loss. A 25 year old male with family history of type 1 diabetes An obese female with recurrent vaginitis A 50 year old hyperlipidemic male Question 5 A 72 year old female patient reports a 6 month history of gradually progressive swollen and painful distal interphalangeal (DIP) joints of one hand. She has no systemic symptoms but the erythrocyte sedimentation rate (ESR), antinuclear antibody (ANA), and rheumatoid factor (RF) are all minimally elevated. What is the most likely diagnosis? Rheumatoid arthritis (RA) Osteoarthritis (OA) Lupus Peripheral neuropathy Question 6 A 32 year old male patient complains of urinary frequency and burning on urination for 3 days. Urinalysis reveals bacteriuria. He denies any past history of urinary tract infection. The initial treatment should be: Trimethoprim-sulfamethoxazole for 3 days Ciprofloxacin for 7-10 days Trimethoprim-sulfamethoxazole for 14 days Ciprofloxacin for 3 days A thymectomy is usually recommended in the early treatment of which disease? Parkinson's disease Multiple sclerosis Myasthenia gravis Huntington's chorea Question 8 The diagnosis of human papilloma virus (HPV) infection in males is usually made by Clinical appearance Viral culture Tzanck smear KOH prep Question 9 The most effective intervention(s) to prevent stroke is (are): 81 mg of aspirin daily Carotid endarterectomy for patients with high-grade carotid lesions Routine screening for carotid artery stenosis with auscultation for bruits Smoking cessation and treatment of hypertension Question 10 What is the most common cause of Cushing's syndrome? Excessive ACTH production Administration of a glucocorticoid or ACTH Pituitary adenoma or a non-pituitary ACTH-producing tumor Autonomous cortisol production from adrenal tissue Question 11 Diagnostic radiological studies are indicated for low back pain: Routinely after 3 weeks of low back pain symptoms. To screen for spondylolithiasis in patients less than 20 years of age with 2 weeks of more of low back pain. When there is a suspicion of a space-occupying lesion, fracture, cauda equina, or infection. As a part of a pre-employment physical when heavy lifting is included in the job description. Question 12 After treating a patient for Helicobacter pylori infection, what test do you order to see if it has been cured? An enzyme-linked immunosorbent assay titer A urea breath test A rapid urease test A repeat endoscopy Question 13 Which appropriate test for the initial assessment of Alzheimer's disease provides the performance ratings on 10 complex, higher order activities? MMSE CAGE questionnaire FAQ - Functional Activities Questionnaire Holmes and Rahe social readjustment scale Question 14 Major depression occurs most often in which of the following conditions? A. Myocardial infarction Parkinson's disease Stroke Alzheimer's disease Question 15 Which of the following statements about multiple sclerosis (MS) is correct? MS is a chronic, untreatable illness that is almost always fatal. MS is a disease of steadily progressive and unrelenting neurologic deterioration. MS is a chronic, treatable illness with unknown cause and a variable course. Patients with MS who take active steps to improve their health have the best cure rate. Question 16 Diagnostic evaluation of hypothyroidism reveals: Elevated TSH and decreased T4 Decreased TSH and increased T4 Decreased TSH and decreased T3 Elevated TSH and increased T4 Question 17 Risk factors for prostate cancer include all of the following except: Family history Benign prostatic hypertrophy African American race Age Question 18 What information should patients with diabetes and their families receive about hypoglycemia? Hypoglycemia is a rare complication. Hypoglycemia requires professional medical treatment. Hypoglycemia is serious, dangerous, and can be fatal if not treated quickly. Hypoglycemia occurs only as a result of insulin overdose. Question 19 Which history is commonly found in a patient with glomerulonephritis? Beta-hemolytic strep infection Frequent urinary tract infections Kidney stones Hypotension Question 20 Which of the following is characteristic of a manic episode? weight loss of gain insomnia or hypersomnia diminished ability to think or concentrate grandiose delusions Question 21 Central obesity, "moon" face, and dorsocervical fat pad are associated with: Metabolic syndrome Unilateral pheochromocytoma Cushing's syndrome None of the above Question 22 Which of the following is the most common cause of low back pain? Lumbar disc disease Spinal stenosis Traumatic fracture Osteoporosis Question 23 A middle-aged female presents complaining of recent weight loss. The physical exam reveals an enlarged painless cervical lymph node. The differential diagnosis for this patient's problem includes: Infection Toxoplasmosis Mononucleosis All of the above None of the above Question 24 An 81-year-old female is diagnosed with type 2 diabetes. When considering drug therapy for this patient, the nurse practitioner is most concerned with which of the following side effects? Weight gain Fracture risk Hypoglycemia Weight loss Question 25 A patient has HIV infection and is having a problem with massive diarrhea. You suspect the cause is cryptococcosis toxoplasmosis cryptosporidiosis cytomegalovirus Question 26 Which of the following is the most common causative organism of nongonococcal urethritis? Chlamydia trachomatis Ureaplasma urealyticum Mycoplasma hominis Trichonomas vaginalis Question 27 The most common symptoms of transient ischemic attack (TIA) include Nausea, vomiting, syncope, incontinence, dizziness, and seizure. Weakness in an extremity, abruptly slurred speech, or partial loss of vision, and sudden gait changes. Headache and visual symptoms such as bright spots or sparkles crossing the visual field. Gradual onset of ataxia, vertigo, generalized weakness, or lightheadedness Question 28 What is the first symptom seen in the majority of patients with Parkinson's disease? Rigidity Bradykinesia Rest tremor Flexed posture Question 29 A 21-year-old female presents to the office complaining of urinary frequency and urinary burning. The nurse practitioner suspects a urinary tract infection when the urinalysis reveals 1-4 red blood cells per high-powered field Specific gravity 1.012 Urobilinogen 10- white blood cells per high-powered field Question 30 A 26 year old female presents with elbow pain that is described as aching and burning. There is point tenderness along the lateral aspect of the elbow and painful passive flexion and extension. She reports she has been playing tennis almost daily for the past month. The most likely diagnosis is: Radial tunnel syndrome Ulnar collateral ligament sprain Olecranon bursitis Lateral epicondylitis: Tennis Elbow Question 31 Maria, age 17, was raped when she was 13 year old. She is now experiencing sleeping problems, flashbacks, and depression. What is your initial diagnosis? Depression Panic disorder Anxiety Post-traumatic stress disorder Question 32 An elderly man is started on lisinopril and hydrochlorhiazide for hypertension. Three days later, he returns to the office complaining of left great toe pain. On exam, the nurse practitioner notes an edematous, erythematous tender left great toe. The likely precipitant of this patient's pain is: Trauma Tight shoes Arthritis flare Hydrochlorothiazide Question 33 Which of the following is a contraindication for metformin therapy? Insulin therapy Creatinine > 1.5 Edema None of the above Question 34 A positive drawer sign supports a diagnosis of: Sciatica Cruciate ligament injury Meniscal injury Patellar ligament injury Question 35 Steve, age 69, has gastroesophageal reflux disease (GERD). When teaching him how to reduce his lower esophageal sphincter pressure, which substances do you recommend that he avoid? Apples Peppermint Cucumbers Popsicles Question 36 A patient taking levothyroxine is being over-replaced. What condition is he at risk for? Osteoporosis Constipation Depression Exopthalmia Question 37 Diabetes screening recommendations for asymptomatic adults age 45 and over include which of the following: HbA1C 2-hour 75 gram oral glucose tolerance test C-peptide level A and B All of the above Question 38 Dave, age 38, states that he thinks he has an ear infection because he just flew back from a business trip and feels unusual pressure in his ear. You diagnose barotrauma. What is your next action? Prescribe systemic antibiotics Prescribe antibiotic ear drops Prescribe nasal steroids and oral decongestants Refer him to an ear, nose, and throat specialist Question 39 Josh, age 22, is a stock boy and has an acute episode of low back pain. You order and NSAID and tell him which of the following? Maintain moderate bed rest for 3-4 days Call the office for narcotics if there is no relief with the NSAID in 24-48 hours Begin lower back strengthening exercises depending on pain tolerance Wear a Boston brace at night Question 40 Risk factors for Addison's disease include which of the following? Tuberculosis Autoimmune disease AIDS All of the above Question 41 Urine cultures should be obtained for which of the following patients? Suspected urinary tract infection in pregnancy Febrile patients Young men All of the above Question 42 Jennifer says that she has heard that caffeine can cause osteoporosis and asks you why. How do you respond? "Caffeine has not effect on osteoporosis." "A high caffeine intake has a diuretic effect that may cause calcium to be excreted more rapidly." "Caffeine affects bone metabolism by altering intestinal absorption of calcium and assimilation of calcium into the bone matrix." "Caffeine increase bone resorption." Question 43 The diagnosis which must be considered in a patient who presents with a severe headache of sudden onset, with neck stiffness and fever, is: Migraine headache Subarachnoid hemorrhage Glaucoma Meningitis Question 44 What conditions must be met for you to bill "incident to" the physician, receiving 100% reimbursement from Medicare? You must initiate the plan of care for the patient The physician must be on-site and engaged in patient care You must be employed as an independent contractor You must be the main health care provider who sees the patient Question 45 The most commonly recommended method for prostate cancer screening in a 55 year old male is: Digital rectal examination (DRE) plus prostate specific antigen (PSA) Prostate specific antigen (PSA) alone Transrectal ultrasound (TRUS) alone Prostate specific antigen (PSA) and transrectal ultrasound (TRUS) Question 46 The primary goals of treatment for patients with alcohol abuse disorder are: Reduction in withdrawal symptoms and reduction in desire for alcohol Psychotherapeutic and pharmacological interventions to decrease desire for and effects of alcohol Abstinence or reduction in use, relapse prevention, and rehabilitation Marital satisfaction, improvement in family functioning, and reduction in psychiatric impairment Question 47 You are assessing a patient after a sports injury to his right knee. You elicit a positive anterior/posterior drawer sign. This test indicates an injury to the: Lateral meniscus Cruciate ligament Medical meniscus Collateral ligament Question 48 Marsha presents with symptoms resembling both fibromyalgia and chronic fatigue syndrome, which have many similarities. Which of the following is more characteristic of fibromyalgia? Musculoskeletal pain Difficulty sleeping Depression Fatigue Question 49 Differential diagnosis of proteinuria includes which of the following? Orthostatic proteinuria Nephrotic syndrome Infection Trauma A and B Question 50 A patient presenting for an annual physical exam has a BMI of 25 kg/m2 This patient would be classified as: Underweight Normal weight Overweight Obese Question 51 Reed-Sternberg B lymphocytes are associated with which of the following disorders: Aplastic anemia Hodgkin's lymphoma Non Hodgkin's lymphoma Myelodysplastic syndromes Question 53 A 77-year-old female presents to the office complaining a sudden swelling on her right elbow. She denies fever, chills, trauma, or pain. The physical exam reveals a non-tender area of swelling over the extensor surface over the right elbow with evidence of trauma or irritation. The nurse practitioner suspects: Arthritis Ulnar neuritis Septic arthritis Olecranon bursitis Question 54 Establishment of a definitive diagnosis of osteomyelitis requires: A known causative injury such as a puncture wound, bite, or decubitus ulcer. Biopsy of culture of the pathogen from blood or bone aspirate. Visualization of purulent material draining into soft tissue. Lucent areas identified on plain x-ray. Question 55 The most accurate measure of diabetes control is: Avoidance of micro- and macro-vascular complications. Insulin sensitivity. Early morning glucose levels. HgbA1c Question 56 A patient has been diagnosed with generalized anxiety disorder (GAD). Which of the following medications may be used to treat generalized anxiety disorder? Alprazolam or diazepam Venlafaxine or buspirone Trazodone or sertraline Venlafaxine or hydroxyzine pamoate Question 57 The most common presentation of thyroid cancer is: Generalized enlargement of the thyroid gland. A solitary thyroid nodule. A multinodular goiter. Abnormal thyroid function tests. Question 58 Which of the following accounts for half of the bladder tumors among men and one-third in women? Cigarette smoke, both active and passive inhalation Chemicals from plastic and rubber Chronic use of phenacetin-containing analgesic agents Working long hours and not voiding often Question 59 You have a new patient that presents with generalized lymphadenopathy. You know that this is indicative of: Sjogren's syndrome Pancreatic cancer Disseminated malignancy of the hematologic system Cancer of the liver Question 60 Sally, a computer programmer, has just been given a new diagnosis of carpal tunnel syndrome. Your next step is to: Refer her to a hand surgeon Take a more complete history Try neutral position wrist splinting and oral NSAID Order a nerve conduction study such as am electromyography Question 61 Which drug category contains the drugs that are the first line Gold standard therapy for COPD? Corticosteroids Inhaled beta-2 agonist bronchodilators Inhaled anticholinergic bronchodilators Xanthines Question 62 Potential causes of septic arthritis include which of the following? Lyme disease Prosthetic joint infection Reiter's syndrome A and B All of the above Question 63 The nurse practitioner diagnoses epididymitis in a 24 year old sexually active male patient. The drug of choice for treatment of this patient is: Oral ciprofloxacin (Cipro) Oral doxycycline (Virbamycin) plus intramuscular ceftriaxone Oral trimethoprim-sulfamethoxazole (Bactrim DS) Intramuscular penicillin Question 64 A patient has been taking fluoxetine (Prozac) since being diagnosed with major depression, first episode, 2 months ago. She reports considerable improvement in her symptoms and her intention to discontinue the medication. What should be the nurse practitioner's recommendation? Advise the patient to stop the antidepressant medication Question the patient to determine if the self-assessment is correct before advising her to discontinue the medication Recommend that the patient continue the antidepressant medication for at least 4 more months Discuss with the patient the need to take the antidepressant medication indefinitely Question 65 Your patient has an elevated mean cell volume (MCV). What should you be considering in terms of diagnosis? Iron-deficiency anemia Hemolytic anemia Lead poisoning Liver disease Question 66 Which of the following medications increase the risk for metabolic syndrome? Antihistamines Proton pump inhibitors Protease inhibitors A and C All of the above Question 67 The hallmark of neurofibromatosis (von Recklinghausen's disease) present in almost 100% of patients is Acoustic neuroma Astrocytoma of the retina Distinctive osseous lesions Café au lait spots Question 68 The most commonly recommended pharmacological treatment regimen for low back pain (LBP) is: Acetaminophen or an NSAID A muscle relaxant as an adjunct to an NSAID An oral corticosteroid and diazepam (Valium) Colchicine and an opioid analgesic Question 69 Legal authority for advanced practice nursing rests with: The Health Care Financing Administration Federal statutes State laws and regulations Certifying bodies Question 70 Which of the following physical modalities recommended for treatment of rheumatoid arthritis provides the most effective long term pain relief? Superficial and deep heat Application of cold Transcutaneous electrical nerve stimulation (TENS) Exercise Question 71 A patient presents with dehydration, hypotension, and fever. Laboratory testing reveals hyponatremia, hyperkalemia, and hypoglycemia. These imbalances are corrected, but the patient returns 6 weeks later with the same symptoms of hyperpigmentation, weakness, anorexia, fatigue, and weight loss. What action(s) should the nurse practitioner take? Obtain a thorough history and physical, and check serum cortisol and ACTH levels. Obtain a diet history and check CBC and FBS. Provide nutritional guidance and have the patient return in 1 month. Consult home health for intravenous administration of fluids and electrolytes. Question 72 Jack, age 55, comes to the office with a blood pressure of 144/98 mm Hg. He states that he did not know if it was ever elevated before. When you retake his blood pressure at the end of the exam, it remains at 144/98. What should your next action be? Start him on an ACE Inhibitor Start him on a diuretic Have him monitor his blood pressure at home Try nonpharmacological methods and have him monitor his blood pressure at home Martin is complaining of erectile dysfunction. He also has a condition that has reduced arterial blood flow to his penis. The most common cause of this condition is: Parkinson's disease Epilepsy Multiple sclerosis Diabetes mellitus Question 74 Which of the following is not a risk factor associated with the development of syndrome X and type 2 diabetes mellitus? Hypertriglyceridemia and low high-density lipoprotein (HDL) Gestational diabetes and polycystic ovarian syndrome Hispanic, African-American, Native-American, and Pacific Islander ethnicity Postprandial hypoglycemia Question 75 A 28-year-old female presents to the office requesting testing for diagnosis of hereditary thrombophilia. Her father recently had a deep vein thrombosis and she is concerned about her risk factors. The nurse practitioner explains that: The patient should start anticoagulant therapy immediately. Hereditary thrombophilia does not always require anticoagulation therapy. Women of childbearing age cannot take anticoagulant therapy. Genetic and risk management counseling are recommended. B and D Question 76 The 4 classic features of Parkinson's disease are Mask-like facies, dysarthria, excessive salivation, and dementia. Tremor at rest, rigidity, bradykinesia, and postural disturbances. Depression, cognitive impairment, constipation and shuffling gait. Tremor with movement, cogwheeling, repetitive movement, and multi-system atrophy. Question 77 Which of the following set of symptoms should raise suspicion of a brain tumor? Recurrent, severe headaches that awaken the patient and are accompanied by visual disturbances. Vague, dull headaches that are accompanied by a reported sense of impending doom. Periorbital headaches occurring primarily in the evening and accompanied by pupillary dilation and photophobia. Holocranial headaches present in the morning and accompanied by projective vomiting without nausea. Question 78 The most common cause of elevated liver function tests is: Hepatitis Biliary tract obstruction Chronic alcohol abuse A drug-induced injury Question 79 The physiological explanation of syncope is: Accelerated venous return and increased stroke volume resulting in deactivation of the parasympathetic nervous system. A cycle of inappropriate vasodilation, bradycardia, and hypotension. A sudden rise in blood pressure due to overly efficient vasoconstriction. Emotional stress resulting in hypertension, tachycardia, and increased venous return. Question 80 A 60 year old male patient with multiple health problems presents with a complaint of erectile dysfunction (ED). Of the following, which medication is most likely to be causing the problem? Thiazide diuretic Insulin Famotidine (Pepcid) Albuterol Question 81 The cardinal sign of infectious arthritis is: Affected joint is painful at rest, with movement and weight bearing Rapid onset that wakes the patient during the night Long history of severe pain with associated joint swelling None of the above Question 82 A 75-year-old female is diagnosed with primary hyperparathyroidism and asks the nurse practitioner what the treatment for this disorder is. The nurse practitioner explains: Primary hyperparathyroidism is treated with Vitamin D restriction Primary hyperparathyroidism is treated with parathyroidectomy Primary hyperparathyroidism is treated with daily magnesium Primary hyperparathyroidism is treated with parenteral parathyroid hormone (PTH) Question 83 Which of the following is not appropriate suppression therapy for chronic bacterial prostatitis? Doxycycline 100 mg qd Nitrofurantoin 100 mg qd Bactrim DS qd Erythromycin qd Question 84 Who is at a higher risk for developing nephrolithiasis? Jack, who exercises every day and drinks copious amounts of water Mary, who watches her weight and eats a low-sodium diet Harvey, a "couch potato" who drinks a lot of no-sodium soda Bill, who runs every day and takes excessive amounts of vitamin C Question 85 Potential side effects of levofloxacin include which of the following? Confusion Hypoglycemia Achilles tendon rupture All of the above Question 86 A patient has just been diagnosed with Bell's palsy. He is understandably upset and has questions about the prognosis. You response should be: Although most of your symptoms will disappear, some will remain but can usually be camouflaged by altering your hairstyle or growing a beard Unfortunately there is no cure but you have a mild case The condition is self-limiting and most likely complete recovery will occur With suppressive drug therapy you can minimize the symptoms Question 87 A diabetic patient is taking low-dose enalapril for hypertension. A record of the patient's blood pressure over 4 weeks ranges from 130 to 142 mmHg systolic and 75 to 85 mmHg diastolic. How should the nurse practitioner respond? Change to a different class of antihypertensive medication to get better control. Increase the dosage of the current BP medication. Continue the current medication and dosage for 4 more weeks. Add a beta-blocker to the current medication regimen. What diabetic complications result from hyperglycemia? 1. Retinopathy 2. Hypertension resistant to treatment 3. Peripheral neuropathy 4.Accelerated atherogenesis 1, 2, 3 2, 3, 4 1, 3, 4 1, 2, 4 Question 89 Diagnostic confirmation of acute leukemia is based on: Bone marrow aspiration and biopsy Pancytopenia Hyperuricemia All of the above Question 90 The best test to determine microalbuminuria to assist in the diagnosis of diabetic neuropathy: A dipstick strip done during routine urinalysis in the office A 24-hour urine collection An early morning spot urine collection A serum albumin test Question 91 Which of the following antibiotics should not be prescribed for a pregnant woman in the 3rd trimester? Trimethoprim-sulfamethoxazole Erythromycin Cefuroxime Levofloxacin Question 92 Martin, a 58 year old male with diabetes, is at your office for his diabetes follow up. On examining his feet with monofilament, you discover that he has developed decreased sensation in both feet. There are no open areas or signs of infection on his feet. What health teaching should Martin receive today regarding the care if his feet? Wash your feet with cold water daily See a podiatrist every 2 years, inspect your own feet monthly, and apply lotion to your feet daily Go to a spa and have a pedicure monthly See a podiatrist yearly; wash your feet daily with warm soapy water and towel dry between the toes; inspect your feet daily for lesions; apply lotion to dry areas Question 93 The intervention known to be most effective in the treatment of severe depression, with or without psychosis, is: Psychotherapy Electroconvulsive therapy (ECT) A selective serotonin reuptake inhibitor (SSRI) A tricyclic antidepressant (TCA) Question 94 The initial clinical sign of Dupuytren's contracture is: Pain with ulnar deviation Painless nodule on palmer fascia Pain and numbness in the ring finger Inability to passively extend finger Question 95 Which of the following is the most common complication of the myelodysplastic syndromes Fatigue Cardiomyopathy Falls Bleeding Question 96 What information should a 42 year old patient with newly diagnosed diabetes receive about exercise? Buy good walking shoes with support and a flexible sole. Exercise at least 5 days per week. Snack before exercise. Do not exercise if your blood sugar is greater than 180 mg/dL Question 97 What is the most commonly abused substance? Heroin Cocaine Alcohol Marijuana Question 98 Sam, age 67, is a diabetic with worsening renal function. He has frequent hypoglycemic episodes, which he believes means that his diabetes is getting "better." How do you respond? "You're right; it seems like your diabetes is improving." "Because your kidneys are not functioning well, your insulin is not being metabolized and excreted as it should, so you need less of it." "You have watched your diet for all these years and as a result, your body is using less insulin." "I will have to change your oral hyperglycemic agents as it seems your body is making more insulin." Question 99 A child with type 1 diabetes mellitus has experienced excessive hunger, weight gain and increasing hyperglycemia. The Somogyi effect is suspected. What steps should be taken to diagnose and treat this condition? Decrease the evening insulin dose and check capillary blood glucose (CBG) at 2:00 am. Instruct the child's parents on physical activities to help weight loss. Increase the evening insulin dose and check CBG at 2:00 am. Refer the child for instruction on a strict diabetic diet. Question 100 At what age is screening most likely to detect scoliosis? 4 to 6 years 8 to 10 years 12 to 14 years 18 to 20 years Question 101 The most reliable indicator(s) of neurological deficit when assessing a patient with acute low back pain is(are): Patient report of bladder dysfunction, saddle anesthesia, and motor weakness of limbs. History of significant trauma relative to the patient's age. Decreased reflexes, strength, and sensation in the lower extremities. Patient report of pain with the crossed straight leg raise. A patient who has had a swollen, nontender scrotum for one week is found to have a mass within the tunica vaginalis that transilluminates readily. The family nurse practitioner suspects: a.) a hydrocele. b.) a varicocele. c.) an indirect inguinal hernia. d.) carcinoma of the testis. a hydrocele. A client had excessive blood loss and prolonged hypotension during surgery. His postoperative urine output is sharply decreased, and his blood urea nitrogen (BUN) is elevated. The most likely cause for the change is acute: A) Prerenal inflammation B) Bladder outlet obstruction C) Tubular necrosis D) Intrarenal nephrotoxicity Bladder outlet obstruction Mr. S. comes to you with scrotal pain. The examinations of his scrotum, penis, and rectum are normal. Which of the following conditions outside of the scrotum may present as scrotal pain? A. Inguinal herniation and peritonitis B. Renal colic and cardiac ischemia C. Pancreatitis and Crohn ' s disease D. Polyarteritis nodosa and ulcerative colitis Inguinal herniation and peritonitis The most common type of hernia is a(n): A. indirect inguinal hernia. B. direct inguinal hernia. C. femoral hernia. D. umbilical hernia. indirect inguinal hernia. Max, age 70, is obese. He is complaining of a bulge in his groin that has been there for months. He states that it is not painful, but it is annoying. You note that the origin of swelling is above the inguinal ligament directly behind and through the external ring. You diagnose this as a(n): A. indirect inguinal hernia. B. direct inguinal hernia. C. femoral hernia. D. strangulated hernia. direct inguinal hernia. A 35 year old sexually active man presents with a 1 week history of fever and pain over the left scrotum. It is accompanied by frequency and dysuria. The scrotum is edematous and tender to touch. He denies flank pain, nausea, and vomiting. He reports that eh pain is lessend when he uses scrotal-support briefs. The urinalysis shows 2 + blood and a large number of leukocytes. What is the most likely diagnosis? A. Acute urinary tract infection B. Acute pyelonephritis C. Acute orthitis D. Acute epididymitis Acute epididymitis Orchitis is caused by which of the following? A. Mumps virus B. Measles virus C. Chlamydia trachomatis D. Chronic urinary tract infections that are not treated adequately Mumps virus A 10 year old boy complains of sudden onset of scrotal pain upon awakening that morning. He is also complaining of severe nausea and vomiting. During the physical examination, the nurse practitioner finds a tender, warm, and swollen left scrotum. The cremastic reflex is negative and the urine dipstick is negative for leukocytes, nitrites, and blood. The most likely diagnosis is: A. Acute epididymitis B. Severe salmonella infection C. Testicular torsion D. Acute orchitis Testicular torsion What type of follow up should this patient receive? A. Refer to a urologist within 48 hours B. Refer him to the emergency department as soon as possible C. Prescribe ibuprofen (advil) 600 mg QID for pain D. Order a testicular ultrasound for further evaluation Refer him to the emergency department as soon as possible A 24-year-old man presents with sudden onset of left-sided scrotal pain. He reports having intermittent unilateral testicular pain in the past but not as severe as this current episode. Confirmation of testicular torsion would include all of the following findings except: A. unilateral loss of the cremasteric reflex. B. the affected testicle held higher in the scrotum. C. testicular swelling. D. relief of pain with scrotal elevation. relief of pain with scrotal elevation In assessing a man with testicular torsion, the NP is most likely to note: A. elevated PSA level. B. white blood cells reported in urinalysis. C. left testicle most often affected. D. increased testicular blood flow by color-flow Doppler ultrasound. left testicle most often affected. Anticipated organ survival exceeds 85% with testicular decompression within how many hours of torsion? A. 1 B. 6 C. 16 D. 24 6 To prevent a recurrence of testicular torsion, which of the following is recommended? A. use of a scrotal support B. avoidance of testicular trauma C. orchiopexy D. limiting the number of sexual partners orchiopexy Jordan appears with a rapid onset of unilateral scrotal pain radiating up to the groin and flank. You are trying to differentiate between epididymitis and testicular torsion. Which test to determine whether swelling is in the testis or the epididymis should be your first choice? A. X-ray B. Ultrasound C. Technetium scan D. Physical examination Ultrasound The nurse practitioner recognizes that the most common cause of epididymitis in a young man is: A chlamydia B E. coli C mycoplasma D Proteus species Chlamydia Your 25-year-old male patient has had a fever, dysuria, low back pain, and scrotal edema. Which of the following is likely the diagnosis? A acute bacteria prostatitis B acute pyelonephritis C epididymitis D urinary tract infection epididymitis The action of a 5 alpha-reductace inhibitor in the treatment of BPH is to: reduce action of androgens in the prostate. Milton, a 72 year old unmarried, sexually active white man presents to your clinic with complaints of hesitancy, urgency, and occasional uncontrolled dribbling. Although you suspect benign prostatic hypertrophy, what else should your differential diagnosis include? Urethral stricture (may develop as a result of sexually transmitted diseases and should be considered in a sexually active individual no matter what the age) Harry has BPH and complains of some incontinence. Your first step in diagnosing overflow incontinence would be to order a: Post void residual urine measurement Lower urinary tract symptoms in males can present as a constellation of storage or voiding symptoms. Storage symptoms include: urgency and nocturia A 63-year-old man presents to you with hematuria, hesitancy, and dribbling. DRE reveals a moderately enlarged prostate that is smooth. The client's PSA is 1.2. What is the most appropriate management strategy for you to follow at this time? Prescribe an alpha adrenergic blocker, which will relax bladder and prostate smooth muscle to improve flow and relieve symptoms. According to the AUA guideline on the management of BPH, when is referral for invasive surgery automatically warranted? With the presence of refractory retention and bladder stones. What differentiates prostate cancer symptoms from BPH? Symptoms of prostate cancer in general tend to progress more rapidly than those of BPH. The most common gram-negative bacteria that causes both acute and chronic bacterial prostatitis is A. Staphylococcus aureus B. Klebsiella C. Escherichia coli D. Enterobacteriaceae Escherichia coli A history of urinary tract infections in males is often seen in men with chronic bacterial prostatitis. Other signs and symptoms of chronic bacterial prostatitis includes A. irritative voiding symptoms, low back pain, and perineal pain. B. nausea and vomiting, as well as fever. C. loss of appetite and weight loss. D. irritative voiding symptoms, inability to ambulate, and fever. irritative voiding symptoms, low back pain, and perineal pain. When performing a prostate examination, you note a tender, warm prostate. What do you suspect? A. Benign prostatic hypertrophy B. Prostatic abscess C. Prostate cancer D. Bacterial prostatitis Bacterial prostatitis Gerard is complaining of a scrotal mass; however, the scrotum is so edematous that it is difficult to assess. How do you determine if it is a hernia or a hydrocele? Bowel sounds may be heard over a hernia. Mr. S comes to you with scrotal pain. The examinations of his scrotum, penis, and rectum are normal. Which of the following conditions outside of the scrotum may present as scrotal pain? Inguinal hernia and peritonitis A 17 year old boy reports feeling something on his left scrotum. On palpation, soft and movable blood vessels that feel like a "bag of worms" are noted underneath the scrotal skin. It is not swollen or reddened. The most likely diagnosis is A. Chronic orchitis B. Chronic epididymitis C. Testicular torsion D. Varicocele Varicocele What risk factors contribute to varicocele? A. Younger age B. Current cigarette smoker C. Multiple sex partners D. None of the above None of the above Treatment options for varicocele repair include all of the following except: A. Open surgery B. Laparoscopic surgery C. Treatment with a thrombolytic agent D. Percutaneous embolization Treatment with a thrombolytic agent There is a higher risk of balanitis in which of the following conditions? A. Renal Insufficiency B. Diabetes Mellitus C. Graves' disease D. Asthma Diabetes Mellitus Balanitis is caused by: A. Staphylococcus aureus B. Streptococcus pyogenes C. Candida albicans D. Trichomonads Candida albicans Balanitis is a symptom of which one of the following diseases A. Psoriatic arthritis B. Reactive arthritis C. Alkylosing Spondylitis D. Rheumatoid arthritis Reactive arthritis The average American man has an approximately % lifetime risk of prostate cancer and an approximately % likelihood of clinical disease. A. 15, 5 B. 25, 8 C. 40, 10 D. 60, 15 40, 10 All of the following can cause an elevated PSA level except: A. Current prostate infection B. Recent cystoscopy C. BPH D. Prostatectomy Prostatectomy According to recent epidemiologic studies, prostate cancer is the number cause of cancer death in men residing within the US. (B) A. 1 B. 2 C. 3 D. 4 2 You perform a DRE on a 72-year old man and find a lesion suspicious for prostate cancer. The findings are described as: A. a rubbery, enlarged prostatic lobe. B. an area of prostatic induration C. a boggy gland. D. prostatic tenderness an area of prostatic induration All of the following can cause a elevated PSA level except: A. current prostate infection B. recent cystoscopy C. BPH D. prostatectomy prostatectomy A 54 year old white man with no obvious risk for prostate cancer opted to undergo PSA screening and DRE testing. The DRE findings are normal and his PSA is 3..7ng/ml. You recommend: A. Repeating the PSA test immediately. B. Repeat screening in 1 yr. C. Repeat screening in 2 yrs. D. Repeat screening in 5 yrs. Repeat screening in 1 yr. Which of the following mediations is associated with the highest incidence of erectile dysfunction? a. Lamotrigine (Lamictal) b. Clonazepam (Klonopin) c. Paroxetine (Paxil) d. Doxepin (Sinequan) Paroxetine (Paxil) Which of the following is not a common risk factor for erectile dysfunction (ED)? A. diabetes mellitus B. hypertension C. cigarette smoking D. testosterone deficiency testosterone deficiency When taking a phosphodiesterase-5 (PDE-5) inhibitor, concomitant use of which medication must be avoided? A. statins B. sulfonylurea C. angiotensin-converting enzyme (ACE) inhibitors D. nitrates nitrates What percent of patients with genital warts have spontaneous regression of the lesions? a. 10% B. 50% C. 25% D. 75% 50% Which HPV types are the most likely to cause genital warts or condyloma acuminatum? A. 1,2, & 3 B. 16 & 18 C. 6 & 11 D. 22 & 24 6 & 11 Treatment for patients with condyloma acuminatum include all of the following except: A. Topical acyclovir B. Podofilox C. cryotherapy D. trichloroacetic acid Topical acyclovir What antibiotic would you expect NOT to give to a pregnant woman with a UTI? Bactrim - TMP avoid use during pregnancy secondary to possible risk of teratogenicity What is the most common symptom of upper UTI in young children? Fever What lab result will be altered if a patient has taken an antibiotic? Urine Culture What is the preferred method to collect a urine specimen in children who are not toilet-trained? catheterization When would you consider a CT scan or renal ultrasound for patients with acute pyelonephritis? If the patient has not improved after 48-72 hours on an appropriate antibiotic. What four reasons would you consider the infection complicated until proven otherwise? If the patient is a man, an older adult, or child, or has symptoms lasting more than 7 days What medication would you expect not to use in acute pyelonephritis? Do not use nitrofurantoin because poor tissue concentrations are achieved in the renal parenchyma To decrease the production of uric acid stones, the family nurse practitioner orders which medication? A. allopurinol (zyloprim) B. indomethacin (indocin) C. bethanechol (urecholine) D. phenazopyridine (pyridium) allopurinol (zyloprim) You see a 58-year-old man diagnosed with a kidney stone who reports pain primarily during urination. You consider all of the following except: A. improved hydration. B. alpha blocker use. C. prescribing a diuretic. D. analgesia use prescribing a diuretic. Your 35-year-old patient is being worked up for microscopic hematuria. All of the following are differential diagnoses of microscopic hematuria except: A) Kidney stones B) Bladder cancer 527 C) Acute pyelonephritis D) Renal artery stenosis Renal artery stenosis A 65-year-old carpenter complains of stiffness and pain in both hands and right knee shortly after waking and worsens in the afternoon. He feels some relief with rest. On, exam the nurse practitioner notices the presence of Heberden's nodes. Which of the following is most likely? a. Osteoporosis b. Rheumatoid Arthritis c. Osteoarthritis d. Reiter's syndrome Osteoarthritis Mary, age 72, has severe osteoarthritis of her right knee. She obtains much relief from corticosteroid injections. When she asks you how often she can have them, how do you respond? A. Only once a year in the same joint B. No more than twice a year in the same joint C. No more than three to four times a year in the same joint D. No more than five to six times a year in the same joint No more than three to four times a year in the same joint Your 75-year-old client with osteoarthritis of the knee will be starting on a course of NSAIDs for pain management. The most important teaching point for your patient currently is: A. You should start with a high dose first and taper down the dose as needed. B. You should continue to take your Coumadin as you have been. C. Report any excessive stomach upset or if you notice that your stools become dark or bloody. D. At this point, it will not be helpful to lose weight. Report any excessive stomach upset or if you notice that your stools become dark or bloody. A 45-year-old woman in complaining of generalized stiffness, especially in both her wrist and hands. It is much worse in the morning and last a few hours. She also complains of fatigue and generalized body aches that have been present for the past few months. Which of the following is most likely? a. Osteoporosis b. Rheumatoid arthritis c. Osteoarthritis d. Gout Rheumatoid arthritis Ginny, age 48, has rheumatoid arthritis and gets achy and stiff after sitting through a long movie. This is referred to as A. longevity stiffness B. gelling C. intermittent arthritis D. molding gelling Mrs. Matthews has rheumatoid arthritis. On reviewing an x-ray of her hip, you notice that there is a marked absence of articular cartilage. What mechanism is responsible for this? A. Antigen-antibody formation B. Lymphocyte response C. Immune complex formation D. Lysosomal degradation Lysosomal degradation Karina a client with myasthenia gravis is to receive immunosuppressive therapy. The nurse understands that this therapy is effective because it: A. Stimulates the production of acetylcholine at the neuromuscular junction. B. Promotes the removal of antibodies that impair the transmission of impulses C. Inhibits the breakdown of acetylcholine at the neuromuscular junction. D. Decreases the production of autoantibodies that attack the acetylcholine receptors. Decreases the production of autoantibodies that attack the acetylcholine receptors. Treatment for Myasthenia Gravis Includes: A. IVIG or Plasmapheresis B. Antibiotics C. Analgesic's D. Sedation IVIG or Plasmapheresis What specific treatment is given patients diagnosed with Myasthenia Gravis that will focus on improving conduction? A. Surgery B. Physical Therapy C. Mestinon (pyridostigmine) Neostigmine D. Fluroquinolones Mestinon (pyridostigmine) Neostigmine First-line treatment of SLE in a patient with mild symptoms is: A. systemic corticosteroids. B. hydroxychloroquine plus NSAIDs. C. anakinra. D. methotrexate. hydroxychloroquine plus NSAIDs. Common physical findings of SLE include all the following except: A. weight gain. B. joint pain and swelling. C. fatigue. D. facial rash. weight gain. Which of the following is not characteristic of rheumatoid arthritis (RA)? A. It is more common in women than in men at a 3:1 ratio. B. Family history of autoimmune conditions often is reported. C. Peak age for disease onset in individuals is age 50 to 70 years. D. Wrists, ankles, and toes often are involved. Peak age for disease onset in individuals is age 50 to 70 years. The use of all of the following medications can trigger gout except: A. aspirin. B. statins. C. diuretics. D. niacin. statins. Which of the following dietary supplements is associated with increased risk for gout? A. vitamin A B. gingko biloba C. brewer's yeast D. glucosamine brewer's yeast Which of the following conditions places a patient at an increased risk of plantar fasciitis? A. Diabetes B. Pregnancy C. Alcoholism D. Thyroid disease Diabetes A 48-year old obese female presents to the clinic with complain of pain on the bottom of her feet with the first few steps in the morning that gets worse with prolonged walking. You would suspect: A. Gout B. Morton's Neuroma C. Rheumatoid Arthritis D. Plantar fasciitis Plantar fasciitis Which of the following is used to evaluate patients with medial collateral ligament injury? a. Varus test b. McMurray test c. Lachman test d. Valgus test Lachman test The Lachman maneuver is used to detect which of the following? A. instability of the knee B. nerve damage of the knee due to past knee injuries C. integrity of the patellar tendon D. tears on the meniscus of the knee tears on the meniscus of the knee A positive drawer sign supports a diagnosis of: a. sciatica b. cruciate ligament injury c. hip dislocation d. patellar ligament injury cruciate ligament injury A 15-year-old high school athlete complains of a painful area below both knees. He tells you they feel bone-like and are tender to palpation. He denies any hot joints, fever, rash, or difficulties with weight- bearing. Which of the following conditions is most likely? a) osteomyelitis b) internal tibial torsion c) Osgood-Schlatter Disease d) Calve-Perthe's Disease Osgood-Schlatter Disease When caring for the child with Osgood-Schlatter Disease, the nurse practitioner would know her treatment has been effective when: a. The child no longer complains of pain at lower knee at rest b. The child no longer complains of pain at the ankle at rest. c. The child no longer complains of pain at the hip during exercise. d. The child no longer complains of pain at lower knee during exercise. The child no longer complains of pain at lower knee during exercise. Common risk factors for Osgood Schlatter Disease include: a. Highly active with open growth plates b. Significant Growth Spurt c. Typically asymmetrical but can occur bilaterally d. Boys> Girls e. All of the above All of the above What must you you do to diagnose a distal femur, patellar, and proximal tibia fracture? Obtain imaging according to Ottawa knee rules What are the key physical findings of patellar fractures? palpable DEFECT on patella, HEMARTHROSIS, Failure to do STRAIGHT LEG RAISE = failure of EXTENSOR MECHANISM (retinaculum displaced) Heidi, age 29, is a nurse who has an acute episode of back pain. You have determined that it is a simple "mechanical" backache and order A) Bedrest for 2 days B) Muscle relaxants C) "Let pain be your guide" and continue activities D) Back-strengthening exercises "Let pain be your guide" and continue activities Beth, age 49, comes in with low back pain. An x-ray of the lumbar/sacral spine is within normal limits. Which of the following diagnoses do you explore further? A. Scoliosis B. Osteoarthritis C. Spinal stenosis D. Herniated nucleus pulposus Herniated nucleus pulposus John, age 17, works as a stock boy at the local supermarket. He is in the office for a routine visit. You notice that he had an episode of low back pain 6 months ago from improperly lifting heavy boxes. In discussing proper body mechanics with him to prevent future injuries, you tell him, A. " Bend your knees and face the object straight on. " B. " Hold boxes away from your body at arm 's length. " C. " Bend and twist simultaneously as you lift. " D. " Keep your feet firmly together. " " Bend your knees and face the object straight on. " With the straight-leg-raising test, the NP is evaluating tension on which of the following nerve roots? A. L1 and L2 B. L3 and L4 C. L5 and S1 D. S2 and S3 L5 and S1 The most common sites for lumbar disk herniation are: A. L1 to L2 and L2 to L3. B. L2 to L3 and L4 to L5. C. L4 to L5 and L5 to S1. D. L5 to S1 and S1 to S2 L4 to L5 and L5 to S1 Immediate diagnostic imaging for low back pain should be reserved for all of the following EXCEPT: A. presence of signs of the cauda equina syndrome. B. presence of severe neurological deficits. C. presence of risk factors for cancer. D. presence of moderate pain lasting at least 2 weeks. presence of moderate pain lasting at least 2 weeks. First-line therapy for prepatellar bursitis should include: A. bursal aspiration. B. intrabursal corticosteroid injection. C. acetaminophen. D. knee splinting. bursal aspiration. Clinical conditions with a presentation similar to acute bursitis include which of the following? (More than one can apply.) A. rheumatoid arthritis B. septic arthritis C. joint trauma D. pseudogout all of the above Patients with subscapular bursitis typically present with: A. limited shoulder ROM. B. heat over affected area. C. localized tenderness under the superomedial angle of the scapula. D. cervical nerve root irritation localized tenderness under the superomedial angle of the scapula. Which of the following is usually NOT part of treatment of a sprain? A. immobilization B. applying ice to the area C. joint rest D. local corticosteroid injection local corticosteroid injection If any limitation or any increase in range of motion occurs when assessing the musculoskeletal system, the angles of the bones should be measured by using A. Phalen ' s test. B. skeletometry. C. the Thomas test. D. a goniometer. a goniometer. In assessing the skeletal muscles, you turn the forearm so that the palm is up. This is called A. supination. B. pronation. C. abduction. D. eversion. supination. The most common cause of acute bursitis is: A. Inactivity B. Joint overuse C. Fibromyalgia D. Bacterial infection Joint overuse First-line treatment options for bursitis usually include: A. corticosteroid bursal injection. B. heat to area. C. weight-bearing exercises. D. nonsteroidal anti-inflammatory drugs (NSAIDs). nonsteroidal anti-inflammatory drugs (NSAIDs). Likely sequelae of intrabursal corticosteroid injection include: A. irreversible skin atrophy. B. infection. C. inflammatory reaction. D. soreness at the site of injection. soreness at the site of injection. Patients with lateral epicondylitis typically present with: a. electric-like pain elicited by tapping over the median nerve b. reduced joint pain c. pain that is worst with elbow flexion (elbow ROM is usually normal) d. decreased hand grip strength the pain is worse with resisted wrist extension decreased hand grip strength the pain is worse with resisted wrist extension Risk factors for lateral epicondylitis include all of the following EXCEPT: A. repetitive lifting B. playing tennis C. hammering D. gout gout Up to what percentage of patients with medial epicondylitis recover without surgery? A. 35% B. 50% C. 70% D. 95% 95% Initial treatment of lateral epicondylitis includes all of the following EXCEPT: A. rest and activity modifications B. corticosteroid injections C. topical or oral NSAIDS D. counterforce bracing brace centered over the back of the forearm can help relieve symptoms corticosteroid injections Carpal tunnel syndrome is inflammation of the: A) Ulnar nerve B) Radial nerve C) Brachial nerve D) Median nerve Median nerve The Phalen test is used to evaluate for: A) Inflammation of the median nerve B) Rheumatoid arthritis C) Degenerative joint changes D) Chronic tenosynovitis Inflammation of the median nerve Sandra, a computer programmer, has just been given a new diagnosis of carpal tunnel syndrome. Your next step is to A. refer her to a hand surgeon. B. take a more complete history. C. try neutral position wrist splinting and order an oral NSAID. D. order a nerve conduction study such as an electromyography (EMG) try neutral position wrist splinting and order an oral NSAID. Alexander, age 18, sprained his ankle playing ice hockey. He is confused as to whether to apply heat or cold. What do you tell him? A. " Use continuous heat for the first 12 hours, then use heat or cold to your own preference. " B. " Use continuous cold for the first 12 hours, then use heat or cold to your own preference. " C. " Apply cold for 20 minutes, then take it off for 30 - 45 minutes; repeat for the first 24 - 48 hours while awake. " D. " Alternate between cold and heat for 20 minutes each for the first 24 - 48 hours. " " Apply cold for 20 minutes, then take it off for 30 - 45 minutes; repeat for the first 24 - 48 hours while awake. " When Maxwell, age 12, slid into home plate while playing baseball, he injured his ankle. You are trying to differentiate between a sprain and a strain. You know that a sprain A. is an injury to the ligaments that attach to bones in a joint. B. is an injury to the tendons that attach to the muscles in a joint. C. is an injury resulting in extensive tears of the muscles. D. does not result in joint instability. is an injury to the ligaments that attach to bones in a joint. Which of the following is usually not a part of treatment of a sprain? A. Immobilization B. Applying ice to the area C. Joint rest D. Local corticosteroid injection Local corticosteroid injection Janice is recovering from osteomyelitis of her leg. She asks you for advice as to what she can do to promote healing. You tell her to: A. put weight on the affected leg more frequently to promote increased circulation, oxygenation, and nutrition to the tissues of the wound area. B. eat foods high in vitamins and calcium and increase her calorie and protein intake. C. spend time in the fresh air and expose the wound to fresh air and sunlight. D. be sure to use strict aseptic technique when changing the dressing, which should be kept wet at all times to improve wound healing. eat foods high in vitamins and calcium and increase her calorie and protein intake. Dan, age 49, developed osteomyelitis of the femur after a motorcycle accident. Which of the following statements about the clinical manifestations of osteomyelitis is correct? A. Integumentary effects include swelling, erythema, and warmth at the involved site. B. There is a low-grade fever with intermittent chills C. Musculoskeletal effects include tenderness of the entire leg. D. Cardiovascular effects include bradycardia. Integumentary effects include swelling, erythema, and warmth at the involved site. What is the protein lacking or absent in muscular dystrophy a. coronin b. tubulin c. dystrophin d. keratin dystrophin Type of muscular dystrophy that is most common in adulthood a. Becker md b. Duchenne md c. facioscapulohumeral md d. myotonic md. myotonic md. What type of muscular dystrophy do females get? A. becker md b. Duchenne md c. myotonic md d. none none The following are causes of scoliosis except: a. unknown b. spinal congenital deformity c. arthritis d. herniated disc. herniated disc. It is defined as a true deformity of the spine a. functional scoliosis b. structural scoliosis c. arthritic scoliosis d. neurofibroma scoliosis structural scoliosis Screening for females are recommended at what age? A. 5 and 12 B. 7 and 11 3. 14 and 15 d. 10 and 12. 10 and 12. Bouchard's nodule is found in which of the following? A) Rheumatoid arthritis B) Degenerative joint disease C) Psoriatic arthritis D) Septic arthritis Degenerative joint disease Podagra is associated with which of the following? A) Rheumatoid arthritis B) Gout C) Osteoarthritis D) Septic arthritis Gout Which of the following statements is most consistent with fibromyalgia? A. It is predominantly diagnosed in African Americans. B. It affects less than 1% of the general population. C. It is four to seven times more common in women than in men. D. It is most often initially diagnosed in adults younger than 20 years old and older than 55 years old. It affects less than 1% of the general population. Fibromyalgia is more common in patients with: A. type 2 diabetes. B. rheumatoid arthritis and systemic lupus erythematosus. C. migraine headaches. D. chronic obstructive pulmonary disorder (COPD). rheumatoid arthritis and systemic lupus erythematosus. Which of the following is inconsistent with the clinical presentation of fibromyalgia? A. widespread body aches B. joint swelling C. fatigue D. cognitive changes fatigue Jonas, age 62, experienced a temporary loss of consciousness that was associated with an increased rate of respiration, tachycardia, pallor, perspiration, and coolness of the skin. How would you describe this? A. Lethargy B. Delirium C. Syncope D. A fugue state Syncope The Hallpike maneuver is performed to elicit A. a seizure. B. vertigo. C. syncope. D. a headache. syncope. Common signs of high-grade aortic stenosis in an individual during exercise include all of the following except: A. dyspnea. B. angina. C. seizure. D. syncope. syncope. All of the following are possible etiologies for papilledema a. Intracranial abscess b. Ruptured aneurysm c. Migraine headaches d. Cerebral edema Migraine headaches The following measures are used for treating various types of headache. Which is not considered to be effective therapy for migraine? a. Propanolol prophylaxis b. Resting in a quiet and darkened room c. Trimethobenzamide suppositories for nausea d. Sodium restriction to decrease water retention Sodium restriction to decrease water retention Which of the following is indicated in the prophylactic treatment of migraine headache a. ibuprofen (Motrin) b. Naproxen sodium (Anaprox) c. Propanolol (Inderal) d. Sumatriptan (Imitrex) Propanolol (Inderal) An adult woman comes to the clinic presenting with gradual onset of throbbing headaches behind one eye worsening over several eyes is diagnosed as a. Cluster headache b. Migraine headache c. Trigeminal neuralgia d. Temporal arteritis Migraine headache In tension-type headache, which of the following statements is true? a. Photophobia is seldom reported. b. The pain is typically described as "pressing" in quality c. The headache is usually unilateral d. Physical activity usually makes the discomfort worse The pain is typically described as "pressing" in quality A first-line prophylactic treatment option for the prevention of tension-type headache is a. Nortriptyline b. Verapamil c. Carbamazepine d. valproate Nortriptyline Migraine associated with muscle or neck pain, which is NOT an International Headache Society migraine diagnostic criterion, is often diagnosed as a. Sinus headache b. Cluster headache c. Increased intracranial pressure d. Tension-type headache Tension-type headache Acute cerebral hemorrhage is best identified with which of the following imaging techniques? a. transesophageal echocardiogram b. CT Scan c. Cerebral angiogram d. MR angiography CT Scan A 64-year-old woman presents to your clinic with a sudden right-sided headache that is worse in her right eye. She claims her vision seems blurred, and her right pupil is dilated and slow to react. The right conjunctiva is markedly injected, and the eyeball is firm. You screen her vision and find that she is 20/30 OS and 20/30 OD. She most likely has A. open-angle glaucoma. B. angle-closure glaucoma. C. herpetic conjunctivitis. D. diabetic retinopathy. angle-closure glaucoma. Sylvia has glaucoma and has started taking a medication that acts as a diuretic to reduce the intraocular pressure. Which medication is she taking? A. A carbonicanhydrase inhibitor B. A beta-adrenergic receptor blocker C. A miotic D. A mydriatic A carbonicanhydrase inhibitor Signs and symptoms of acute angle-closure glaucoma include A. painless redness of the eyes. B. loss of peripheral vision. C. translucent corneas. D. halos around lights. halos around lights. When evaluating the person who has bacterial meningitis, the NP expects to find CSF results of: a. low protein b. predominance of lymphocytes c. glucose at about 30% of serum levels d. low opening pressure glucose at about 30% of serum levels An 18-year-old college freshman is brought to the student health center with a chief complaint of a 3-day history of progressive headache and intermittent fever. On physical examination he has a positive Kernig and Brudzinski sign. The most likely diagnosis is: a. viral encephalitis b. bacterial meningitis c. acute subarachnoid hemorrhage d. epidural hematoma bacterial meningitis Which of the following diseases is associated with a high risk of giant cell arteritis? A. history of transiet ischemia attacks (TIA) B. Frequent migraine headaches with focal neurological findings C. Polymalgia rheumatica (PMP) D. Systemic lupus erythematous (SLE) Polymalgia rheumatica (PMP) Which of the following diagnostic tests would be most helpful in the diagnosis of this illness? A. CT scan of the brain B. Cranial nerve exam C. Sedimentation rate D. CBC with differential Sedimentation rate Which of the following conditions is most likely? A. cluster headache B. migraine headache with aura C. migraine headache without aura D. Giant cell arteritis Giant cell arteritis Of the four types of strokes, which one is the most common and has a gradual onset? A. Thrombotic B. Embolic C. Lacunar D. Hemorrhagic Thrombotic Which of the following risk factors for a stroke can be eliminated? A. Hypertension B. Carotid artery stenosis C. Smoking D. Hyperlipidemia Smoking Which of the following complications is the leading cause of death shortly after a stroke? A. Septicemia B. Pneumonia C. Pulmonary embolus D. Ischemic heart disease Pneumonia Risk factors for transient ischemic attack (TIA) include all of the following except: A. atrial fibrillation. B. carotid artery disease. C. combined oral contraceptive use. D. pernicious anemia. pernicious anemia. A TIA is characterized as an episode of reversible neurological symptoms that can last: A.1 hour. B.6 hours. C.12 hours. D.24 hours 24 hours When caring for a patient with a recent TIA, you consider that: A. long-term antiplatelet therapy is likely indicated. B. this person has a relatively low risk of future stroke. C. women present with this disorder more often than men. D. rehabilitation will be needed to minimize the effects of the resulting neurological insult. long-term antiplatelet therapy is likely indicated. A 56-year-old man complains of several episodes of severe lacerating pain that shoots up to his right cheek and is precipitated by drinking cold drinks or chewing. These episodes start suddenly and end spontaneously after a few seconds with several episodes per day. He denies any trauma, facial weakness, or difficulty swallowing. He has stopped drinking cold drinks because of the pain. Which of the following is most likely? A. Trigeminal neuralgia B. Cluster headache C. Acute sinusitis D. Sinus headache Trigeminal neuralgia Which of the following cranial nerves is evaluated when a wisp of cotton is lightly brushed against the corner of the eye? A. CN II B. CN III C. CN IV D. CN V CN V You examine a 29-year-old woman who has a sudden onset of right sided facial asymmetry. She is unable to close her right eyelid tightly, frown, or smile on the affected side. Her exam is otherwise unremarkable. This presentation likely represents paralysis of CN: A: lll B. lV C. Vll D. Vlll Vll Which represents the most appropriate diagnostic test for the patient in the previous question? A. CBC with WBC Differential? B. Lyme disease antibody titer? C. CT scan of the head with contrast? D. BUN and Creatine levels? Lyme disease antibody titer? In prescribing prednisone for the patient with Bells palsy, the NP considers that its use: A. Has not been shown to be helpful in improving outcomes in the condition. B. Should be initiated asap after the onset of facial paralysis. C. Is likely to help minimize ocular symptoms. D. May prolong the course of the disease. Should be initiated asap after the onset of facial paralysis. Treatment options for an adult with seizures include all of the following agents EXCEPT: a. Carbamazepine b. Phenytoin c. Gabapentin d. tamsulosin tamsulosin Which of the following best describes patient presentation during tonic-clonic (grand mal) seizure? a. Blank staring lasting 3 to 50 seconds, accompanied by impaired level of consciousness b. Awake state with abnormal motor behavior lasting seconds c. Rigid extension of arms and legs, followed by sudden jerking movements with loss of consciousness d. Abrupt muscle contraction with autonomic signs Rigid extension of arms and legs, followed by sudden jerking movements with loss of consciousness Central obesity, “moon” face, and dorsocervical fat pad are associated with: An elderly man is started on lisinopril and hydrochlorhiazide for hypertension. Three days later, he returns to the office complaining of left great toe pain. On exam, the nurse practitioner notes an edematous, erythematous tender left great toe. The likely precipitant of this patient’s pain is: The most effective treatment of non-infectious bursitis includes: Conservative treatment includes rest, cold and heat treatments, elevation, administration of nonsteroidal anti-inflammatory drugs (NSAIDs), bursal aspiration, and intrabursal steroid injections What conditions must be met for you to bill “incident to” the physician, receiving 100% reimbursement from Medicare? Selected Answer: The physician must be on-site and engaged in patient care Answers: You must initiate the plan of care for the patient The physician must be on-site and engaged in patient care You must be employed as an independent contractor You must be the main health care provider who sees the patient Which of the following is not a risk factor associated with the development of syndrome X and type 2 diabetes mellitus? The metabolic syndrome refers to the co-occurrence of several known cardiovascular risk factors, including insulin resistance, obesity, atherogenic dyslipidemia and hypertension. Which of the following is not a common early sign of benign prostatic hyperplasia (BPH)? A. Nocturia B. Urgency incontinence C. Strong urinary stream flow D. Straining to void Steve, age 69, has gastroesophageal reflux disease (GERD). When teaching him how to reduce his lower esophageal sphincter pressure, which substances do you recommend that he avoid? Food that is very hot or very cold Fatty or fried foods Peppermint or spearmint, including flavoring Coffee, tea, and soft drinks that contain caffeine Spicy, highly seasoned foods Fried food DT caffeine, chocolate and anticholinergics Which drug category contains the drugs that are the first line Gold standard therapy for COPD? Beta antagonist The most commonly recommended pharmacological treatment regimen for low back pain (LBP) is: Nsaid Which of the following is not appropriate suppression therapy for chronic bacterial prostatitis? Erythromycin A patient presents with dehydration, hypotension, and fever. Laboratory testing reveals hyponatremia, hyperkalemia, and hypoglycemia. These imbalances are corrected, but the patient returns 6 weeks later with the same symptoms of hyperpigmentation, weakness, anorexia, fatigue, and weight loss. What action(s) should the nurse practitioner take? .A Obtain a thorough history and physical, and check serum cortisol and ACTH levels. B. Perform a diet history and check CBC and FBS. C. Provide nutritional guidance and have the patient return in one month. D. Consult home health for intravenous administration You are assessing a patient after a sports injury to his right knee. You elicit a positive anterior/posterior drawer sign. This test indicates an injury to the: he A. lateral meniscus B. cruciate ligament C. medial meniscus D. collateral ligament. A 32 year old female patient presents with fever, chills, right flank pain, right costovertebral angle tenderness, and hematuria. Her urinalysis is positive for leukocytes and red blood cells. The nurse practitioner diagnoses pyelonephritis. The most appropriate management is: Include 500 mg of oral ciprofloxacin (Cipro) twice per day for seven days; 1,000 mg of extended-release ciprofloxacin once per day for seven days; or 750 mg of levofloxacin (Levaquin) once per day for five days. A middle-aged man presents to urgent care complaining of pain of the medial condyle of the lower humerus. The man works as a carpenter and describes a gradual onset of pain. On exam, the medial epicondyle is tender and pain is increased with flexion and pronation. Range of motion is full The most likely cause of this patient’s pain is: epicondylitis The best test to determine microalbuminuria to assist in the diagnosis of diabetic neuropathy s to measure albumin in a spot urine sample, collected either as the first urine in the morning or at random, for example, at the medical visit. This method is accurate,:Early morning What is the first symptom seen in the majority of patients with Parkinson’s disease? Tremor at rest The most commonly recommended method for prostate cancer screening in a 55 year old male is: digital rectal exams, Martin, age 24, presents with an erythematous ear canal, pain, and a recent history of swimming. What do you suspect? Otitis externa/ swimmer’s ear Which of the following symptoms suggests a more serious cause of back pain? Pain associated with lying down at night A patient taking levothyroxine is being over-replaced. What condition is he at risk for? osteoporosis Which of the following is the most common cause of low back pain? Which is the most common cause of end-stage renal disease in the United States? Diabetes A 77-year-old female presents to the office complaining a sudden swelling on her right elbow. She denies fever, chills, trauma, or pain. The physical exam reveals a non-tender area of swelling over the extensor surface over the right elbow with evidence of trauma or irritation. The nurse practitioner suspects: A 60 year old female patient complains of sudden onset unilateral, stabbing, surface pain in the lower part of her face lasting a few minutes, subsiding, and then returning. The pain is triggered by touch or temperature extremes. Physical examination is normal. Which of the following is the most likely diagnosis? A. fibromyligia or neuro something Beth, age 49, comes in with low back pain. An x-ray of the lumbosacral spine is within normal limits. Which of the following diagnoses do you explore further? Arthritis, C • Question 34 • D 1 out of 1 points A patient exhibits extrapyramidal side effects of antipsychotic medications. Which of the following symptoms would lead you to look for another diagnosis? high fever (102 to 104 degrees F), irregular pulse, accelerated heartbeat (tachycardia), increased rate of respiration (tachypnea), muscle rigidity, altered mental status, autonomic nervous system dysfunction resulting in high or low blood pressure, profuse perspiration, and excessive sweating. called Malignant Hyperthermia” OR Neuroleptic malignant syndrome hallucinations. Phalen’s test, 90°wrist flexion for 60 seconds, reproduces symptoms of: carpal tunnel syndrome The most common cause of elevated liver function tests is: alcohol Reed-Sternberg B lymphocytes are associated with which of the following disorders: A Aplastic anemia . B Hodgkin’s lymphoma . C Non Hodgkin’s . lymphoma D Myelodysplastic . syndromes Which of the following is a potential acquired cause of thrombophilia? A Homocysteinuria . B Protein C deficiency . C Factor V Leiden . D Antiphospholipid . antibodies A 75-year-old female is diagnosed with primary hyperparathyroidism and asks the nurse practitioner what the treatment for this disorder is. The nurse practitioner explains: A 25 year old overweight patient presents with a complaint of dull achiness in his groin and history of a palpable lump in his scrotum that “comes and goes”. On physical examination, the nurse practitioner does not detect a scrotal mass. There is no tenderness, edema, or erythema of the scrotum, the scrotum does not transilluminate. What is the most likely diagnosis? A. Testicular torsion B. Epididymitis C. Inguinal hernia D. Varicocele Dave, age 38, states that he thinks he has an ear infection because he just flew back from a business trip and feels unusual pressure in his ear. You diagnose barotrauma. What is your next action? A. Prescribe nasal steroids and oral decongestants B. Prescribe antibiotic eardrops C. Prescribe systemic eardrops D. Refer David to an ear, nose, and throat specialist A. Prescribe The physiological explanation of syncope is: Syncope is a transient loss of consciousness (TLOC) due to global cerebral hypoperfusion characterized by rapid onset, short duration and spontaneous complete recovery. A 20 year old male patient complains of “scrotal swelling.” He states his scrotum feels heavy, but denies pain. On examination, the nurse practitioner notes transillumination of the scrotum. What is the most likely diagnosis? hydrocele A 32 year old male patient complains of urinary frequency and burning on urination for 3 days. Urinalysis reveals bacteriuria. He denies any past history of urinary tract infection. The initial treatment should be: nclude nitrofurantoin monohydrate/macrocrystals, trimethoprim-sulfamethoxazole (TMP-SMX), or fosfomycin. trimethoprim and sulfamethoxazole Diagnostic radiological studies are indicated for low back pain: Diagnostic imaging is indicated for patients with low back pain only if they have severe progressive neurologic deficits or signs or symptoms that suggest a serious or specific underlying condition. In other patients, evidence indicates that routine imaging is not associated with clinically meaningful benefits but can lead to harms. An 81-year-old female is diagnosed with type 2 diabetes. When considering drug therapy for this patient, the nurse practitioner is most concerned with which of the following side effects? The diagnosis of human papilloma virus (HPV) infection in males is usually made by: The diagnosis of HPV in men is made when external genital warts are seen. The diagnosis of genital warts is made by examination of the lesions Which history is commonly found in a patient with glomerulonephritis? upper respiratory tract infection or a skin infection that was caused by those bacteria A patient complains of generalized joint pain and stiffness associated with activity and relieved with rest. This patient history is consistent with which of the following disorders? Osteoarthritis The most common presentation of thyroid cancer is: is an asymptomatic thyroid mass or a nodule that can be felt in the neck. The obligatory criteria for diagnosis of muscular dystrophy (MD) are: Genetic testing to evaluate missing or repeated mutations in the dystrophin gene. A lack of the dystrophin gene can lead to a diagnosis of Duchenne or Becker MD. The test is important not only to confirm the MD diagnosis in males but also to determine whether The case for genetic diagnosis. Accurate diagnosis of the muscular dystrophies is important for patients, their families, and efficient and cost-effective use of medical resources. (EEG.) The diagnosis which must be considered in a patient who presents with a severe headache of sudden onset, with neck stiffness and fever, is: viral meningitis A 60 year old male patient with multiple health problems presents with a complaint of erectile dysfunction (ED). Of the following, which medication is most likely to be causing the problem? Hypertensive medication such as hydrochlorothiazides, and other DM2 medications A 72 year old patient exhibits sudden onset of fluctuating restlessness, agitation, confusion, and impaired attention. This is accompanied by visual hallucinations and sleep disturbance. What is the most likely cause of this behavior? A. Dementia B. Delirium C. Parkinson's disease D. Depression Which of the following set of symptoms should raise suspicion of a brain tumor? Holocranial headaches present in the morning and accompanied by projectile vomiting without nausea The cornerstone of treatment for stress fracture of the femur or metatarsal stress fracture is: relative rest-that being the cessation of the offending activity for a period of time until the irritation is eliminated, pain is gone and the bone is given a chance to heal. Rest the affected part of the body - Once pain free, the person can gradually begin the sporting activity again - Air splinting can reduce pain and decrease the time until return to full participation or intensity of exercise. - NSAIDs to reduce pain and inflammation Sally, a computer programmer, has just been given a new diagnosis of carpal tunnel syndrome. Your next step is to: a. refer her to a hand surgeon b. take a more complete hx c. try neutral position wrist splinting and order an oral NSAID d. order a nerve conduction study such as an electromyography (EMG) Marsha presents with symptoms resembling both fibromyalgia and chronic fatigue syndrome, which have many similarities. Which of the following is more characteristic of fibromyalgia? A. Musculosckeletal pain B. Difficulty sleeping C. Depression D. Fatigue The cardinal sign of infectious arthritis is: Affected joint is painful at rest, with movement and weight bearing Diagnostic evaluation for urinary calculi includes: Diagnosis is based on urinalysis and radiologic imaging, usually noncontrast helical CT. A 15 year-old female patient is 5 feet tall and weighs 85 pounds. You suspect anorexia and know that the best initial approach is to: Having the client in view of staff for 90 minutes after each meal Provide the client with a feeling of responsibility and control over her behavior A 63-year-old man presents to the office with hematuria, hesitancy, and dribbling. Digital rectal exam (DRE) reveals a moderately enlarged prostate that is smooth. The PSA is 1.2. What is the most appropriate management strategy for you to follow at this time? A. Prescribe an alpha adrenergic blocker. B. Recommend saw palmetto. C. Prescribe an antibiotic D. Refer the client to urology. A positive drawer sign supports a diagnosis of: Posterior cruciate ligament injury A 14 year old female cheerleader reports gradual and progressive dull anterior knee pain, exacerbated by kneeling. The nurse practitioner notes swelling and point tenderness at the tibial tuberosity. X-ray is negative. What is the most likely diagnosis? prepatellar bursitis? Jack, age 55, comes to the office with a blood pressure of 144/98 mm Hg. He states that he did not know if it was ever elevated before. When you retake his blood pressure at the end of the exam, it remains at 144/98. What should your next action be? Answer: stage 2 hypertension (lifestyle change, and medication Martin is complaining of erectile dysfunction. He also has a condition that has reduced arterial blood flow to his penis. The most common cause of this condition is: diabetes mellitus Successful management of a patient with attention deficit hyperactivity disorder (ADHD) may be achieved with: Ritalin and diet What diabetic complications result from hyperglycemia? Retinopathy Hypertension resistant to treatment 3. Peripheral neuropathy 4. Accelerated atherogenesis In which of the following presentations is further diagnostic testing not warranted? The most common symptoms of transient ischemic attack (TIA) include: Common symptoms are sudden and transient, and include unilateral paresis, speech disturbance, and monocular blindness. clinical symptoms typically lasting less than one hour, and without evidence of infarction on imaging Establishment of a definitive diagnosis of osteomyelitis requires: Bone biopsy leads to a definitive diagnosis . Which of the following is the most common causative organism of nongonococcal urethritis? A 30 year old female patient presents to the clinic with heat intolerance, tremors, nervousness, and weight loss inconsistent with increased appetite. Which test would be most likely to confirm the suspected diagnosis? TSH. We also assess free thyroid hormone (FT4) and at times T3 Potential causes of hypocalcemia include which of the following? Hypoalbuminemia is the most common cause of hypocalcemia. Causes include cirrhosis, nephrosis . hypoparathyroidism, vitamin D deficiency, and renal disease. Which of the following is a contraindication for metformin therapy? hypersensitity metabolic acidiosis, dehydration, sepsis, hypoxemia, hepatic impairment, renal dysfunction The organism most often associated with prostatitis is: The most effective intervention(s) to prevent stroke is (are): Aspirin is one of the most common, most effective/or life style changes What is the most commonly abused substance? alcohol is the #1 abused substance in America. An estimated 135.5 million people drink alcohol, but of those people, 86 million are considered to be abusers, An obese hyperlipidemic patient, newly diagnosed with type 2 diabetes mellitus, has fasting glucose values 180 to 250 mg/Dl. What is the most appropriate initial treatment to consider? A. A low-calorie diet and exercise B. Sliding-scale NPH insulin every 12 hours C. A sulfonylurea and/or metformin (Glucophage® -XR) D. Sliding-scale regular insulin every 6 hours • Question 100 Prolonged PT suggests: A Platelet abnormality . B Abnormality in intrinsic coagulation . pathway C Abnormality in extrinsic coagulation . pathway D None of the above . Deficiency of FII, FV, FX, or fibrinogen abnormalities A patient presenting for an annual physical exam has a BMI of 25 kg/m2 This patient would be classified as: Overweight • Question 2 1 out of 1 points A patient presents with dehydration, hypotension, and fever. Laboratory testing reveals hyponatremia, hyperkalemia, and hypoglycemia. These imbalances are corrected, but the patient returns 6 weeks later with the same symptoms of hyperpigmentation, weakness, anorexia, fatigue, and weight loss. What action(s) should the nurse practitioner take? Answer: Obtain a thorough history and physical, and check serum cortisol and ACTH levels. Question 5 1 out of 1 points Which of the following is a potential acquired cause of thrombophilia? Antiphospholipid antibodies Question 8 1 out of 1 points Differential diagnosis of proteinuria includes which of the following? • Question 10 1 out of 1 points Question 35 The nurse practitioner diagnoses epididymitis in a 24 year old sexually active male patient. The drug of choice for treatment of this patient is: Ceftriaxone 250 mg IM in a single dose plus. Doxycycline 100 mg orally twice a day for 10 days • Question 11 1 out of 1 points How do you respond when Jessica, age 42, asks you what constitutes a good minimum cardiovascular workout? 20 3x per week • Question 12 1 out of 1 points The intervention known to be most effective in the treatment of severe depression, with or without psychosis, is: electroconvulsive therapy (ECT). • Question 21 1 out of 1 points Which of the following accounts for half of the bladder tumors among men and one- third in women? Cigarette smoke, both active and passive inhalation • Question 23 1 out of 1 points A 21-year-old female presents to the office complaining of urinary frequency and urinary burning. The nurse practitioner suspects a urinary tract infection when the urinalysis reveals Answer: 10WBC/HPF • Question 26 1 out of 1 points The initial clinical sign of Dupuytren’s contracture is: Answer: Painless nodule on palmer fascia • Question 33 1 out of 1 points A nurse practitioner diagnoses a 60 year old male with balanitis. Which disease is commonly associated with balanitis? ( • Question 34 Josh, age 22, is a stock boy and has an acute episode of low back pain. You order and NSAID and tell him which of the following? Begin lower back strengthening exercises depending on pain tolerance. • Question 35 1 out of 1 points A 72 year old female patient reports a 6 month history of gradually progressive swollen and painful distal interphalangeal (DIP) joints of one hand. She has no systemic symptoms but the erythrocyte sedimentation rate (ESR), antinuclear antibody (ANA), and rheumatoid factor (RF) are all minimally elevated. What is the most likely diagnosis? Osteoarthritis • Question 40 1 out of 1 points Jennifer says that she has heard that caffeine can cause osteoporosis and asks you why. How do you respond? "A high caffeine intake has a diuretic effect that may cause calcium to be excreted more rapidly." • Question 41 1 out of 1 points Which of the following set of symptoms should raise suspicion of a brain tumor? Holocranial headaches present in the morning and accompanied by projectile vomiting without nausea • Question 43 1 out of 1 points Diagnostic confirmation of acute leukemia is based on: Bone marrow aspiration and biopsy • Question 44 1 out of 1 points Which of the following antibiotics should not be prescribed for a pregnant woman in the 3rd trimester? Trimethoprim-sulfamethoxazole • Question 46 1 out of 1 points Who is at a higher risk for developing nephrolithiasis? Bill, who runs every day and takes excessive amounts of vitamin C • Question 53 1 out of 1 points A 28-year-old female presents to the office requesting testing for diagnosis of hereditary thrombophilia. Her father recently had a deep vein thrombosis and she is concerned about her risk factors. The nurse practitioner explains that: B and D (does not always require anticoagulation therapy and genetic and risk management counseling are recommended • Question 54 1 out of 1 points Martin, a 58 year old male with diabetes, is at your office for his diabetes follow up. On examining his feet with monofilament, you discover that he has developed decreased sensation in both feet. There are no open areas or signs of infection on his feet. What health teaching should Martin receive today regarding the care if his feet? See a podiatrist yearly; wash your feet daily with warm, soapy water and towel dry between the toes; inspect your feet daily for any lesions; and apply lotion to any dry areas. • Question 57 1 out of 1 points Potential causes of septic arthritis include which of the following? Lyme disease and prosthetic joint infection • Question 58 1 out of 1 points Which of the following is the best response to a woman who has just admitted she is a victim of spousal abuse? "I am concerned about your safety." • Question 62 1 out of 1 points A patient has been diagnosed with generalized anxiety disorder (GAD). Which of the following medications may be used to treat generalized anxiety disorder? benzodiazepines include Xanax, Librium, Valium, and Ativan • Question 64 1 out of 1 points Sam, age 67, is a diabetic with worsening renal function. He has frequent hypoglycemic episodes, which he believes means that his diabetes is getting “better.” How do you respond? Because your kidneys are not functioning well, your insuling is not being metabolized and excreted as it should, so you need less of it • Question 67 1 out of 1 points A 35 year old male presents with a complaint of low pelvic pain, dysuria, hesitancy, urgency, and reduced force of stream. The nurse practitioner suspects acute bacterial prostatitis. Which of the following specimens would be least helpful for diagnosis? Midstream urine specimens for culture and sensitivity CBC and differential Blood urea nitrogen Creatinine • Question 69 1 out of 1 points A patient has just been diagnosed with Bell’s palsy. He is understandably upset and has questions about the prognosis. You response should be: the condition is self-limiting, and most likely complete recovery will occur • Question 70 1 out of 1 points What is the first step in the treatment of uric acid kidney stones? Encouraging hydration • Question 71 1 out of 1 points Urine cultures should be obtained for which of the following patients? All of the above All of the above (pregnant females, males, and febrile patients) • Question 73 1 out of 1 points A patient has been diagnosed with hypothyroidism and thyroid hormone replacement therapy is prescribed. How long should the nurse practitioner wait before checking the patient’s TSH? Answer: 6 – 8 weeks • Question 76 1 out of 1 points Which of the following patients most warrants screening for hypothyroidism? An elderly female with recent onset of mental dysfunction • Question 77 1 out of 1 points The hallmark of neurofibromatosis (von Recklinghausen’s disease) present in almost 100% of patients is: cafe au lait spots. • Question 82 1 out of 1 points A patient presenting for an annual physical exam has a BMI of 25 kg/m2 This patient would be classified as: Overweight • Question 84 1 out of 1 points Diagnostic evaluation of hypothyroidism reveals: Elevated TSH and decrease T4 • Question 85 1 out of 1 points Diagnostic radiological studies are indicated for low back pain: Pain lasting more than 1 month - Objective radiculopathy - Cauda equina syndrome - Considering "red flag": systemic arthritis, infx, malignancy, trauma, congenital/developmental abnormalities • Question 88 1 out of 1 points The correct treatment for ankle sprain during the first 48 hours after injury includes: a cold compress to help reduce swelling (Rest, ice, compression, elevation) • Question 89 1 out of 1 points The most reliable indicator(s) of neurological deficit when assessing a patient with acute low back pain is(are): decreased reflexes, strength, and sensation in the lower extremities • Question 91 1 out of 1 points Risk factors for Addison’s disease include which of the following? Tuberculosis, Autoimmune disease, AIDS (all of the above) • Question 92 1 out of 1 points Major depression occurs most often in which of the following conditions? stroke • Question 93 1 out of 1 points Which of the following medications increase the risk for metabolic syndrome? Answer: antihistamines and proton pump inhibitors, diuretics, beta-blockers • Question 97 1 out of 1 points A 27 year old female patient with epilepsy is well controlled with phenytoin (Dilantin). She requests information about contraception. The nurse practitioner should instruct her that while taking phenytoin: the effective of an oral contraceptive may be reduced • Question 98 1 out of 1 points Risk factors for prostate cancer include all of the following except: BPH • Question 101 1 out of 1 points Maria, age 17, was raped when she was 13 year old. She is now experiencing sleeping problems, flashbacks, and depression. What is your initial diagnosis? Post-traumatic stress disorder Diabetes screening recommendations for asymptomatic adults age 45 and over include which of the following: (HbA1C and 2-hour 75gram oral glucose tolerance test) A child with type 1 diabetes mellitus has experienced excessive hunger, weight gain and increasing hyperglycemia. The Somogyi effect is suspected. What steps should be taken to diagnose and treat this condition? -------------------------------------------- Potential side effects of levofloxacin include which of the following? Confusion, Hypoglycemia, Achilles tendon rupture, All of the above. Which of the following is the most common complication of the myelodysplastic syndromes? Bleeding Which of the following is characteristic of a manic episode? Grandiosity or grandiose delusions Which of the following characteristics are associated with prepatellar bursitis? Repetitive knee trauma A middle-aged female presents complaining of recent weight loss. The physical exam reveals an enlarged painless cervical lymph node. The differential diagnosis for this patient’s problem includes: Answer: infection What is the most common cause of Cushing’s syndrome? the long-term, high-dose use of the cortisol-like glucocorticoids A diabetic patient is taking low-dose enalapril for hypertension. A record of the patient’s blood pressure over 4 weeks ranges from 130 to 142 mmHg systolic and 75 to 85 mmHg diastolic. How should the nurse practitioner respond? Answer: Current guidelines recommend lowering BP to <135/80 mmHg Your patient has an elevated mean cell volume (MCV). What should you be considering in terms of diagnosis? Anemia (pernicious or folic acid anemia) macrocytic Liver disease What information should patients with diabetes and their families receive about hypoglycemia? …………………. At what age is screening most likely to detect scoliosis? Adolescent (age 11 and older, or from onset of puberty until skeletal maturity). A patient has been taking fluoxetine (Prozac) since being diagnosed with major depression, first episode, 2 months ago. She reports considerable improvement in her symptoms and her intention to discontinue the medication. What should be the nurse practitioner’s recommendation? Answer: Recommend that the patient continue the antidepressant medication for at least 4 more months A typical description of a tension headache is: mild to moderate intensity, bilateral, nonthrobbing Which of the following is not a characteristic of type 2 diabetes mellitus? ----------------------------------- Which of the following is not a characteristic of type 1 diabetes mellitus? It is generally controlled by diet and/or oral medication. The most accurate measure of diabetes control is: hemoglobin A1c considered the most accurate measurement or Glycosylated hemoglobin level Microalbuminuria is a measure of: Answer: Urinary excretion of 30-299mg/24 hours of albumin, Expressing albumin and creatinine ratio (ACR) takes the variability into account and normalizes the albumin in a spot specime A patient has HIV infection and is having a problem with massive diarrhea. You suspect the cause is: Cryptosporidiosis The nurse practitioner is following a child with juvenile rheumatoid arthritis (JRA) who has been previously diagnosed and is being managed for the disease by a pediatric rheumatologist. The mother asks for information about the child’s long term prognosis. What is the most appropriate reply? prevent disease progression and destruction of joints, bones, cartilage and soft tissues such as muscles, tendons and joint capsules Which factors are associated with high risk for foot complications in a patient with diabetes mellitus? Obesity Abnormal nails Abnormal gait Poorly controlled lipids Obesity 2.Abnormal nails 3.Abnormal gait 4. Poorly controlled lipids Which of the following is the most common causative organism of nongonococcal urethritis? Chlamydia trachomatis A 65 year old patient complains of recurrent bilateral temporal headaches, malaise, muscle aches, and low grade fever. The headache is described as superficial tenderness rather than deep pain. Giant cell arteritis is suspected. Appropriate treatment is: refer for temporal artery biopsy and initiation of oral prednisone The most reliable diagnostic indicator of gout is: Elevated serum uric acid levels A patient with HIV infection has a fever of unknown origin (FUO). Which of the following is a possible cause of FUO in a patient with HIV? drug fever After treating a patient for Helicobacter pylori infection, what test do you order to see if it has been cured? A urea breath test Which of the following physical modalities recommended for treatment of rheumatoid arthritis provides the most effective long term pain relief? Your physical therapist may use modalities, such as gentle heat and electrical stimulation to help manage your RA symptoms. Other than smoking cessation, which of the following slows the progression of COPD in smokers? oxygen What intervention does the American College of Rheumatology recommend as first line therapy for osteoarthritis? Acetaminophen should be used as first-line therapy for mild osteoarthritis. Nonsteroidal anti-inflammatory drugs are superior to acetaminophen for treating moderate to severe osteoarthritis The best test to determine microalbuminuria to assist in the diagnosis of diabetic neuropathy: Answer: measure albumin (microalbuminuria A 26 year old female presents with elbow pain that is described as aching and burning. There is point tenderness along the lateral aspect of the elbow and painful passive flexion and extension. She reports she has been playing tennis almost daily for the past month. The most likely diagnosis is: lateral epicondylitis (tennis elbow) Janine, age 29, has numerous transient lesions that come and go, and she is diagnosed with urticaria. What do you order? Selected Answer: d. Antihistami nes Answers: a. Aspirin b. NSAIDs c. Opioids d. Antihistami nes Medicare is a federal program administered by the Centers for Medicare and Medicaid Services (CMS). The CMS has developed guidelines for Evaluation and Management coding, which all providers are expected to follow when coding patient visits for reimbursement. Which of the following is an important consideration regarding billing practices? Selected Answer: c. Failing to bill for billable services will lead to unnecessarily low revenues Answers: a. It is important to “undercode” so that one does not get charged with Medicare fraud b. The practice of “overcoding” is essential in this age of decreasing reimbursements c. Failing to bill for billable services will lead to unnecessarily low revenues d. Time spent with the patient is a very important determinant of billing What conditions must be met for you to bill “incident to” the physician, receiving 100% reimbursement from Medicare? Selected Answer: a. You must initiate the plan of care for the patient Answers: a. You must initiate the plan of care for the patient b. The physician must be on-site and engaged in patient care c. You must be employed as an independent contractor d. You must be the main health care provider who sees the patient A 45 year old with diabetes has had itching and burning lesions between her toes for 2 months. Scrapings of the lesions confirm the diagnosis tinea pedis. What is the best initial treatment option for this patient? Selected Answer: Prescribe an antifungal powder for application between her toes and in her shoes and a topical prescription strength antifungal cream for other affected areas. Monitor for a secondary bacterial infection. Answers: Prescribe an antifungal powder for application between her toes and in her shoes and a topical prescription strength antifungal cream for other affected areas. Monitor for a secondary bacterial infection. Prescribe an oral antifungal for 4 to 12 weeks. Monitor BUN and creatinine at 1 week, 2 weeks, and every month thereafter. Prescribe an oral antifungal for 4 to 12 weeks. Monitor liver enzymes, BUN, and creatnine at 1 week, 2 weeks, and every month thereafter. Prescribe a prescription strength antifungal/steroid combination cream. Monitor for a secondary bacterial infection. The Centor criteria for diagnosis of Group A B-hemolytic streptococcus includes which of the following? Selected Answer: A and B only Answers: Fever history Tender, swollen anterior cervical lymph nodes Positive rapid antigen detection test A and B only A, B, and C A 16 year old basketball player complains of itching in the crural folds, buttocks, and upper thighs. The lesions are well demarcated and are half-moon shaped. The area is red, irritated, and there are small breaks in the skin from scratching. What is this patient’s diagnosis and how should it be treated? Selected Answer: a. Tinea cruris; treat with a topical antifungal cream Answers: a. Tinea cruris; treat with a topical antifungal cream b. Eczema; treat with a topical steroid c. Scabies; treat with permethrin cream d. Syphilis; treat with penicillin Stacy, age 27, states that she has painless, white, slightly raised patches in her mouth. They are probably caused by: Selected Answer: c. Candidiasis Answers: a. Herpes simplex b. Aphthous ulcers c. Candidiasis d. Oral cancer Marvin has sudden eye redness that occurred after a strenuous coughing episode. You diagnose a subconjunctival hemorrhage. Your next step is to Selected Answer: c. Do nothing other than provide reassurance Answers: a. Refer him to an ophthalmologist b. Order antibiotics c. Do nothing other than provide reassurance d. Consult with your collaborating physician Your well-nourished 75-year-old patient has come into the office for a physical exam and states that she recently had two nosebleeds. She does not take any anticoagulants, and you have ruled out any coagulopathies. The most likely cause of these nosebleeds is: Selected Answer: b. Trauma or inflammation Answers: a. Sex hormones b. Trauma or inflammation c. A dietary change d. Scurvy Your 31-year-old patient, whose varicella rash just erupted yesterday, asks you when she can go back to work. What do you tell her? Selected Answer: a. Once all the vesicles are crusted over Answers: a. Once all the vesicles are crusted over b. When the rash is entirely gone c. Once you have been on medication for at least 48 hours d. Now, as long as you stay away from children and pregnant women A patient comes in complaining of 1 week of pain in the posterior neck with difficulty turning the head to the right. What additional history is needed? Selected Answer: Any recent trauma Answers: Any recent trauma Difficulty swallowing Stiffness in the right shoulder Change in sleeping habits Which of the following is a predisposing condition for furunculosis? Selected Answer: a. Diabetes mellitus Answers: a. Diabetes mellitus b. Hypertension c. Peripheral vascular disease d. Chronic fatigue syndrome Sylvia, age 83, presents with a 3 day history of pain and burning in the left forehead. This morning she noticed a rash with erythematous papules in that site. What do you suspect? Selected Answer: b. Herpes zoster Answers: a. Varicella b. Herpes zoster c. Syphilis d. Rubella Sarah has allergic rhinitis and is currently being bothered by nasal congestion. Which of the following meds ordered for allergic rhinitis would be most appropriate? Selected Answer: A decongestant nasal spray Answers: An antihistamine intranasal spray A decongestant nasal spray Ipratropium Omalizumab Larry, age 66, is a smoker with hyperlipidemia and hypertension. He is 6 months post-MI. To prevent reinfarction, the most important behavior change that he can make is to: Selected Answer: a. Quit smoking Answers: a. Quit smoking b. Maintain aggressive hypertension therapy c. Stick to a low-fat, low-sodium diet d. Continue with his exercise program Harvey has had Meniere’s disease for several years. He has some hearing loss but now has persistent vertigo. What treatment might be instituted to relieve the vertigo? Selected Answer: c. A vestibular neurectomy Answers: a. Pharmacological therapy b. A labyrinthectomy c. A vestibular neurectomy d. Wearing an earplug in the ear with the most hearing loss A 55 year old man is diagnosed with basal cell carcinoma. The nurse practitioner correctly tells him: Selected Answer: b. “It can be cured with surgical excision or radiation therapy.” Answers: a. “It is the most common cause of death in patients with skin cancer.” b. “It can be cured with surgical excision or radiation therapy.” c. “It is a slow growing skin cancer that rarely undergoes malignant changes.” d. “It can be cured using 5-flurouracil cream twice daily for 2 to 4 weeks.” If a patient presents with a deep aching, red eye and there is no discharge, you should suspect: Selected Answer: a. Iritis Answers: a. Iritis b. Allergic conjunctivitis c. Viral conjunctivitis d. Bacterial conjunctivitis Carl, age 78, is brought to the office by his son, who states that his father has been unable to see clearly since last night. Carl reports that his vision is “like looking through a veil.” He also sees floaters and flashing lights but is not having any pain. What do you suspect? Selected Answer: c. Retinal detachment Answers: a. Cataracts b. Glaucoma c. Retinal detachment d. Iritis Which of the following statements about malignant melanomas is true? Selected Answer: d. The prognosis is directly related to the thickness of the lesion Answers: a. They usually occur in older adult males b. The patient has no family history of melanoma c. They are common in blacks d. The prognosis is directly related to the thickness of the lesion Which drug category contains the drugs that are the first line Gold standard therapy for COPD? Selected Answer: c. Inhaled anticholinergic bronchodilators Answers: a. Corticosteroids b. Inhaled beta-2 agonist bronchodilators c. Inhaled anticholinergic bronchodilators d. Xanthines What condition is associated with mucus production greater than 3 months per year for at least 2 consecutive years? Selected Answer: d. Chronic bronchitis Answers: a. Asthma b. Emphysema c. Chronic obstructive lung disease d. Chronic bronchitis A 57-year-old male presents to urgent care complaining of substernal chest discomfort for the past hour. The EKG reveals ST elevations in Leads II, III, and AVF. The nurse practitioner is aware that these changes are consistent with which myocardial infarction territory? Selected Answer: a. Inferior wall Answers: a. Inferior wall b. Anterior wall c. Apical wall d. Lateral wall Antibiotic administration has been demonstrated to be of little benefit to the treatment of which of the following disease processes? Selected Answer: b. Acute bronchitis Answers: a. Chronic sinusitis b. Acute bronchitis c. Bacterial pneumonia d. Acute exacerbation of chronic bronchitis A patient presents to urgent care complaining of dyspnea, fatigue, and lower extremity edema. The echocardiogram reveals and ejection fraction of 38%. The nurse practitioner knows that these findings are consistent with: Selected Answer: b. Systolic heart failure Answers: a. Mitral regurgitation b. Systolic heart failure c. Cardiac myxoma d. Diastolic heart failure Which of the following best describes hypertrophic cardiomyopathy? Selected Answer: c. Enlarged left ventricle and septum Answers: a. A weakened and enlarged myocardium b. Poor ventricular filling and weakened myocardium c. Enlarged left ventricle and septum d. None of the above Harriet, a 79-year-old woman, comes to your office every 3 months for follow up on her hypertension. Her medications include one baby aspirin daily, Lisinopril 5mg daily, and Calcium 1500 mg daily. At today’s visit. Her blood pressure is 170/89. According to JNC VIII guidelines, what should you do next to control Harriet’s blood pressure? Selected Answer: b. Add a thiazide diuretic to the Lisinopril 5mg daily Answers: a. Increase her Lisinopril to 20mg daily b. Add a thiazide diuretic to the Lisinopril 5mg daily c. Discontinue the Lisinopril and start a combination of ACE Inhibitor and calcium channel blocker d. Discontinue the Lisinopril and start a diuretic Which type of lung cancer has the poorest prognosis? Selected Answer: c. Small cell carcinoma Answers: a. Adenocarcinoma b. Epidermoid carcinoma c. Small cell carcinoma d. Large cell carcinoma Which of the following medication classes should be avoided in patients with acute or chronic bronchitis because it will contribute to ventilation-perfusion mismatch in the patient? Selected Answer: b. Antihistimine s Answers: a. Xanthines b. Antihistimine s c. Steroids d. Anticholinerg ics The most common correlate(s) with chronic bronchitis and emphysema is(are): Selected Answer: b. Cigarette smoking Answers: a. Familial and genetic factors b. Cigarette smoking c. Air pollution d. Occupational environment A patient presents to the office with a blood pressure 142/80. This patient is classified as having: Selected Answer: d. Stage 2 hypertension Answers: a. Normal blood pressure b. Prehypertension c. Stage 1 hypertension d. Stage 2 hypertension Salmeterol (Servent) is prescribed for a patient with asthma. What is the most important teaching point about this medication? Selected Answer: a. It is not effective during an acute asthma attack. Answers: a. It is not effective during an acute asthma attack. b. It may take 2 to 3 days to begin working. c. This drug works within 10 minutes. d. This drug may be used by patients 6 years and older. Other than smoking cessation, which of the following slows the progression of COPD in smokers? Selected Answer: c. Engaging in moderate to high levels of physical activity Answers: a. Making sure the environment is free of all pollutants b. Eliminating all pets from the environment c. Engaging in moderate to high levels of physical activity d. Remaining indoors with air conditioning as much as possible In order to decrease deaths from lung cancer: Selected Answer: d. Patients should be counseled to quit smoking Answers: a. All smokers should be screened annually b. All patients should be screened annually c. Only high risk patients should be screened routinely d. Patients should be counseled to quit smoking An 8 year old presents to the health clinic with history of acute onset severe sore throat and respiratory distress with stridor in the last 2 hours. The child’s history is positive for fever and pharyngitis for 2 days. What is the most likely diagnosis? Selected Answer: a. Mononucleosis Answers: a. Mononucleosis b. Asthma c. Respiratory syncytial virus (RSV) d. Epiglottitis A 20 year old is diagnosed with mild persistent asthma. What drug combination would be most effective in keeping him symptom-free? Selected Answer: d. A bronchodilator PRN and an inhaled corticosteroid Answers: a. A long-acting bronchodilator b. An inhaled corticosteroid and cromolyn c. Theophylline and a short acting bronchodilator d. A bronchodilator PRN and an inhaled corticosteroid Which of the following is not a risk factor for coronary arterial insufficiency? Selected Answer: d. Alcohol ingestion Answers: a. Hyperhomocystein emia b. Smoking c. Genetic factors d. Alcohol ingestion Which of the following patient characteristics are associated with chronic bronchitis? Selected Answer: a. Overweight, cyanosis, and normal or slightly increased respiratory rate Answers: a. Overweight, cyanosis, and normal or slightly increased respiratory rate b. Underweight, pink skin, and increased respiratory rate c. Overweight, pink skin, and normal or slightly increased respiratory rate d. Normal weight, cyanosis, and greatly increased respiratory rate Management of a patient with hypertension and an abdominal aortic aneurysm would include: Selected Answer: b. changing the patient’s BP medications Answers: a. computed tomography scan without contrast b. changing the patient’s BP medications c. referral to a cardiologist d. immediate cardiac catheterization Which of the following is not a goal of treatment for the patient with cystic fibrosis? Selected Answer: d. Replace water-soluble vitamins Answers: a. Prevent intestinal obstruction b. Provide adequate nutrition c. Promote clearance of secretions d. Replace water-soluble vitamins Which of the following patients most warrants screening for hypothyroidism? Selected Answer: d. An elderly female with recent onset of mental dysfunction. Answers: a. A young adult female with postpartum depression lasting 2 weeks. b. A patient taking thyroid replacement preparation. c. A 40 year old male with unexplained tremors. d. An elderly female with recent onset of mental dysfunction. The nurse practitioner diagnoses epididymitis in a 24 year old sexually active male patient. The drug of choice for treatment of this patient is: Selected Answer: b. Oral doxycycline (Virbamycin) plus intramuscular ceftriaxone Answers: a. Oral ciprofloxacin (Cipro) b. Oral doxycycline (Virbamycin) plus intramuscular ceftriaxone c. Oral trimethoprim-sulfamethoxazole (Bactrim DS) d. Intramuscular penicillin The most commonly recommended method for prostate cancer screening in a 55 year old male is: Selected Answer: a. Digital rectal examination (DRE) plus prostate specific antigen (PSA) Answers: a. Digital rectal examination (DRE) plus prostate specific antigen (PSA) b. Prostate specific antigen (PSA) alone c. Transrectal ultrasound (TRUS) alone d. Prostate specific antigen (PSA) and transrectal ultrasound (TRUS) John, age 33, has a total cholesterol level of 188 mg/dL. How often should he be screened for hypercholesterolemia? Selected Answer: a. Every 5 years Answers: a. Every 5 years b. Every 2 years c. Every year d. Whenever blood work is done The National Cholesterol Education Program’s Adult Treatment Panel III recommends that the goal for low density lipoproteins in high risk patients be less than: Selected Answer: c. 100 mg/dL Answers: a. 160 mg/dL b. 130 mg/dL c. 100 mg/dL d. 70 mg/dL A patient presents with classic symptoms of gastroesophageal reflux disease (GERD). He is instructed on life style modifications and drug therapy for 8 weeks. Three months later he returns, reporting that he was “fine” as long as he took the medication. The most appropriate next step is: Selected Answer: d. Investigation with endoscopy, manometry, and/or pH testing Answers: a. Referral for surgical intervention such as a partial or complete fundoplication b. Dependent upon how sever the practitioner believes the condition c. To repeat the 8 week course of drug therapy while continuing lifestyle modifications d. Investigation with endoscopy, manometry, and/or pH testing Which of the following is not appropriate suppression therapy for chronic bacterial prostatitis? Selected Answer: b. Nitrofurantoin 100 mg qd Answers: a. Doxycycline 100 mg qd b. Nitrofurantoin 100 mg qd c. Bactrim DS qd d. Erythromycin qd Of the following choices, the least likely cause of cough is: Selected Answer: c. Acute pharyngitis Answers: a. Asthma b. Gastroesophageal reflux c. Acute pharyngitis d. Allergic rhinitis Which of the following is not a common early sign of benign prostatic hyperplasia (BPH)? Selected Answer: c. Urinary retention Answers: a. Difficulty initiating a urine stream b. Nocturia c. Urinary retention d. Increased force of urine flow A patient has been diagnosed with hypothyroidism and thyroid hormone replacement therapy is prescribed. How long should the nurse practitioner wait before checking the patient’s TSH? Selected d. Answer: 8 weeks Answers: a. 1 week b. 2 weeks c. 4 weeks d. 8 weeks A 20 year old male patient complains of “scrotal swelling.” He states his scrotum feels heavy, but denies pain. On examination, the nurse practitioner notes transillumination of the scrotum. What is the most likely diagnosis? Selected Answer: b. Hydrocele Answers: a. Indirect inguinal hernia b. Hydrocele c. Orchitis d. Testicular torsion Before initiating an HMG CoA-reductase inhibitor for hyperlipidemia, the nurse practitioner orders liver function studies. The patient’s aminotransferase (ALT) is elevated. What laboratory test(s) should be ordered? Selected Answer: a. Serologic markers for hepatitis Answers: a. Serologic markers for hepatitis b. Serum bilirubin c. Serum cholesterol with HDL and LDL d. A liver biopsy What is the most common cause of Cushing’s syndrome? Selected Answer: c. Pituitary adenoma or a non-pituitary ACTH- producing tumor Answers: a. Excessive ACTH production b. Administration of a glucocorticoid or ACTH c. Pituitary adenoma or a non-pituitary ACTH- producing tumor d. Autonomous cortisol production from adrenal tissue Reed-Sternberg B lymphocytes are associated with which of the following disorders: Selected Answer: b. Hodgkin’s lymphoma Answers: a. Aplastic anemia b. Hodgkin’s lymphoma c. Non Hodgkin’s lymphoma d. Myelodysplastic syndromes Which of the following is not a risk factor associated with the development of syndrome X and type 2 diabetes mellitus? Selected Answer: d. Postprandial hypoglycemia Answers: a. Hypertriglyceridemia and low high-density lipoprotein (HDL) b. Gestational diabetes and polycystic ovarian syndrome c. Hispanic, African-American, Native-American, and Pacific Islander ethnicity d. Postprandial hypoglycemia Which of the following are classic features of ulcerative colitis? Selected Answer: d. Remissions and exacerbations of bloody diarrhea, tenesmus, fecal incontinence, abdominal pain and weight loss. Answers: a. Right lower quadrant pain, frequently accompanied by a palpable mass, fever, and leukocytosis. b. Painful hematemesis, occasionally accompanied by melena. c. Rapidly progressive dysphagia with ingestion of solid foods, anorexia, and weight loss out of proportion to the dysphagia. d. Remissions and exacerbations of bloody diarrhea, tenesmus, fecal incontinence, abdominal pain and weight loss. Diagnostic confirmation of acute leukemia is based on: Selected Answer: a. Bone marrow aspiration and biopsy Answers: a. Bone marrow aspiration and biopsy b. Pancytopenia c. Hyperuricemia d. All of the above After thorough history, physical examination, and laboratory tests, a patient is diagnosed with irritable bowel syndrome (IBS). Which of the following initial treatment plans is currently considered most effective? Selected Answer: c. Treatment with a selective serotonin reuptake inhibitor (SSRI) such as fluoxetine (Prozac) or sertraline (Zoloft). Answers: a. A low fat, tyramine-free, caffeine-free, high fiber diet, along with a daily diary, and attention to psychosocial factors. b. Referral to a gastroenterologist for colonoscopy. c. Treatment with a selective serotonin reuptake inhibitor (SSRI) such as fluoxetine (Prozac) or sertraline (Zoloft). d. Antibiotics, nutritional support, and high fiber diet. Steve, age 69, has gastroesophageal reflux disease (GERD). When teaching him how to reduce his lower esophageal sphincter pressure, which substances do you recommend that he avoid? Selected Answer: b. Pepperm int Answers: a. Apples b. Pepperm int c. Cucumb ers d. Popsicles Microalbuminuria is a measure of: Selected Answer: d. Protein lost into the urine. Answers: a. Total urinary protein. b. Late renal compromise in a diabetic patient. c. Early glycemic abnormality. d. Protein lost into the urine. A patient taking levothyroxine is being over-replaced. What condition is he at risk for? Selected Answer: a. Osteoporo sis Answers: a. Osteoporo sis b. Constipati on c. Depressio n d. Exopthal mia The most effective treatment of non-infectious bursitis includes: Selected Answer: b. Rest, an intra-articular corticosteroid injection, and a concomitant oral NSAID. Answers: a. Systemic antibiotic therapy effective against penicillin resistant Staphylococcus areus. b. Rest, an intra-articular corticosteroid injection, and a concomitant oral NSAID. c. A tapering regimen of oral corticosteroid therapy. d. Frequent active range of joint motion. A typical description of a tension headache is: Selected Answer: b. Bilateral, occipital, or frontal tightness or fullness, with waves of aching pain. Answers: a. Periorbital pain, sudden onset, often explosive in quality, and associated with nasal stuffiness, lacrimation, red eye, and nausea. b. Bilateral, occipital, or frontal tightness or fullness, with waves of aching pain. c. Hemicranial pain that is accompanied by vomiting and photophobia. d. Steadily worsening pain that interrupts sleep, is exacerbated by orthostatic changes, and may be preceded by nausea and vomiting. Diagnostic radiological studies are indicated for low back pain: Selected Answer: c. When there is a suspicion of a space-occupying lesion, fracture, cauda equina, or infection. Answers: a. Routinely after 3 weeks of low back pain symptoms. b. To screen for spondylolithiasis in patients less than 20 years of age with 2 weeks of more of low back pain. c. When there is a suspicion of a space-occupying lesion, fracture, cauda equina, or infection. d. As a part of a pre-employment physical when heavy lifting is included in the job description. A 60 year old female patient complains of sudden onset unilateral, stabbing, surface pain in the lower part of her face lasting a few minutes, subsiding, and then returning. The pain is triggered by touch or temperature extremes. Physical examination is normal. Which of the following is the most likely diagnosis? Selected Answer: a. Trigeminal neuralgia Answers: a. Trigeminal neuralgia b. Temporal arteritis c. Parotiditis d. Bell’s palsy A 72 year old patient exhibits sudden onset of fluctuating restlessness, agitation, confusion, and impaired attention. This is accompanied by visual hallucinations and sleep disturbance. What is the most likely cause of this behavior? Selected Answer: b. Delirium Answers: a. Dementia b. Delirium c. Medication reaction d. Depression Phalen’s test, 90°wrist flexion for 60 seconds, reproduces symptoms of: Selected Answer: b. Carpal tunnel syndrome Answers: a. Ulnar tunnel syndrome b. Carpal tunnel syndrome c. Tarsal tunnel syndrome d. Myofascial pain syndrome The most commonly recommended pharmacological treatment regimen for low back pain (LBP) is: Selected Answer: a. Acetaminophen or an NSAID Answers: a. Acetaminophen or an NSAID b. A muscle relaxant as an adjunct to an NSAID c. An oral corticosteroid and diazepam (Valium) d. Colchicine and an opioid analgesic A 26 year old female presents with elbow pain that is described as aching and burning. There is point tenderness along the lateral aspect of the elbow and painful passive flexion and extension. She reports she has been playing tennis almost daily for the past month. The most likely diagnosis is: Selected Answer: d. Lateral epicondylitis Answers: a. Radial tunnel syndrome b. Ulnar collateral ligament sprain c. Olecranon bursitis d. Lateral epicondylitis A positive drawer sign supports a diagnosis of: Selected Answer: b. Cruciate ligament injury Answers: a. Sciatica b. Cruciate ligament injury c. Meniscal injury d. Patellar ligament injury The most effective intervention(s) to prevent stroke is (are): Selected Answer: d. Smoking cessation and treatment of hypertension Answers: a. 81 mg of aspirin daily b. Carotid endarterectomy for patients with high-grade carotid lesions c. Routine screening for carotid artery stenosis with auscultation for bruits d. Smoking cessation and treatment of hypertension The diagnosis which must be considered in a patient who presents with a severe headache of sudden onset, with neck stiffness and fever, is: Selected Answer: d. Meningitis Answers: a. Migraine headache b. Subarachnoid hemorrhage c. Glaucoma d. Meningitis The correct treatment for ankle sprain during the first 48 hours after injury includes: Selected Answer: c. Rest, elevation, compression, ice and pain relief. Answers: a. Alternating heat and ice, and ankle exercises. b. Resistive ankle exercises, ankle support, and pain relief. c. Rest, elevation, compression, ice and pain relief. d. Referral to an orthopedist after x-rays to rule out fracture. Which of the following symptoms suggests a more serious cause of back pain? Selected Answer: c. Pain associated with lying down at night Answers: a. Pain associated with coughing or sneezing b. Pain associated with muscle spasm c. Pain associated with lying down at night d. Pain associated with negative straight leg raise The most reliable indicator(s) of neurological deficit when assessing a patient with acute low back pain is(are): Selected Answer: c. Decreased reflexes, strength, and sensation in the lower extremities. Answers: a. Patient report of bladder dysfunction, saddle anesthesia, and motor weakness of limbs. b. History of significant trauma relative to the patient’s age. c. Decreased reflexes, strength, and sensation in the lower extremities. d. Patient report of pain with the crossed straight leg raise. Which of the following set of symptoms should raise suspicion of a brain tumor? Selected Answer: a. Recurrent, severe headaches that awaken the patient and are accompanied by visual disturbances. Answers: a. Recurrent, severe headaches that awaken the patient and are accompanied by visual disturbances. b. Vague, dull headaches that are accompanied by a reported sense of impending doom. c. Periorbital headaches occurring primarily in the evening and accompanied by pupillary dilation and photophobia. d. Holocranial headaches present in the morning and accompanied by projective vomiting without nausea. Successful management of a patient with attention deficit hyperactivity disorder (ADHD) may be achieved with: Selected Answer: a. Stimulant medication along with behavioral and family intervention. Answers: a. Stimulant medication along with behavioral and family intervention. b. Methylphenidate (Ritalin) in conjunction with diet changes. c. Treatment by a psychiatrist. d. Discipline and removal of offending foods from the diet. Which of the following statements about multiple sclerosis (MS) is correct? Selected Answer: c. MS is a chronic, treatable illness with unknown cause and a variable course. Answers: a. MS is a chronic, untreatable illness that is almost always fatal. b. MS is a disease of steadily progressive and unrelenting neurologic deterioration. c. MS is a chronic, treatable illness with unknown cause and a variable course. d. Patients with MS who take active steps to improve their health have the best cure rate. The most common symptoms of transient ischemic attack (TIA) include: Selected Answer: b. Weakness in an extremity, abruptly slurred speech, or partial loss of vision, and sudden gait changes. Answers: a. Nausea, vomiting, syncope, incontinence, dizziness, and seizure. b. Weakness in an extremity, abruptly slurred speech, or partial loss of vision, and sudden gait changes. c. Headache and visual symptoms such as bright spots or sparkles crossing the visual field. d. Gradual onset of ataxia, vertigo, generalized weakness, or lightheadedness The 4 classic features of Parkinson’s disease are: Selected Answer: b. Tremor at rest, rigidity, bradykinesia, and postural disturbances. Answers: a. Mask-like facies, dysarthria, excessive salivation, and dementia. b. Tremor at rest, rigidity, bradykinesia, and postural disturbances. c. Depression, cognitive impairment, constipation and shuffling gait. d. Tremor with movement, cogwheeling, repetitive movement, and multi-system atrophy. The most common cancer found on the auricle is: Basal Cell Carcinoma Which of the following medication classes should be avoided in patients with acute or chronic bronchitis because it will contribute to ventilation-perfusion mismatch in the patient? Xanthines Antihistimines Steroids Anticholinergics A 47 year old male patient presents to the clinic with a single episode of a moderate amount of bright red rectal bleeding. On examination, external hemorrhoids are noted. How should the nurse practitioner proceed? Instruct the patient on measures to prevent hemorrhoids such as bowel habits and diet. Order a topical hemorrhoid cream along with a stool softener. Refer the patient for a barium enema and sigmoidoscopy. Refer the patient for a surgical hemorrhoidectomy. Which of the following patient characteristics are associated with chronic bronchitis? Overweight, cyanosis, and normal or slightly increased respiratory rate Underweight, pink skin, and increased respiratory rate Overweight, pink skin, and normal or slightly increased respiratory rate Normal weight, cyanosis, and greatly increased respiratory rate A 65-year-old female with a past medical history of hypertension, hyperlipidemia, and polymyalgia rheumatica presents to urgent care with new onset left lower quadrant pain. Her current medications include omeprazole 20 milligrams po daily, lisinopril 20 milligrams po daily, simvastatin 20 milligrams po daily, and prednisone 12 milligrams po daily. The nurse practitioner suspects acute diverticulitis and possibly an abscess. The most appropriate diagnostic test for this patient at this time is: CBC/diff Erythrocyte sedimentation rate Abdominal ultrasound CT scan A patient reports "something flew in my eye" about an hour ago while he was splitting logs. If there were a foreign body in his eye, the nurse practitioner would expect to find all except: Purulent drainage Tearing Photophobia A positive fluorescein stain A 21 year old college student presents to the student health center with copious, markedly purulent discharge from her left eye. The nurse practitioner student should suspect: Viral conjunctivitis Common pink eye Gonococcal conjunctivitis Allergic conjunctivitis A 35 year old man presents with radicular pain followed by the appearance of grouped vesicles consisting of about 15 lesions across 3 different thoracic dermatomes. He complains of pain, burning, and itching. The nurse practitioner should suspect: A common case of shingles and prescribe an analgesic and an antiviral agent A complicated case of shingles and prescribe acyclovir, an analgesic, and a topical cortisone cream Herpes zoster and consider that this patient may be immunocompromised A recurrence of chickenpox and treat the patient's symptoms Which type of lung cancer has the poorest prognosis? Adenocarcinoma Epidermoid carcinoma Small cell carcinoma Large cell carcinoma An 83-year-old female presents to the office complaining of diarrhea for several days. She explains she has even had fecal incontinence one time. She describes loose stools 3-4 times a day for several weeks and denies fever, chills, pain, recent antibiotic use. The history suggests that the patient has: Acute diarrhea Chronic diarrhea Irritable bowel Functional bowel disease Margaret, age 32, comes into the office with painful joints and a distinctive rash in a butterfly distribution on her face. The rash has red papules and plaques with a fine scale. What do you suspect? An allergic reaction Relapsing polychondritis Lymphocytoma cutis Systemic lupus erythematosus Antibiotic administration has been demonstrated to be of little benefit to the treatment of which of the following disease processes? Chronic sinusitis Acute bronchitis Bacterial pneumonia Acute exacerbation of chronic bronchitis Lisa, age 49, has daily symptoms of asthma. She uses her inhaled short-acting beta-2 agonist daily. Her exacerbations affect her activities and they occur at least twice weekly and may last for days. She is affected more than once weekly during the night with an exacerbation. Which category of asthma severity is Lisa in? Mild intermittent Mild persistent Moderate persistent Which of the following is the most appropriate therapeutic regimen for an adult patient with no known allergies diagnosed with group A B-hemolytic strep? Penicillin V 500 milligrams PO every 8 hours for 10 days Ampicillin 250 milligrams PO twice a day for 10 days Clarithromycin 500 milligrams po daily for 7 days None of the above A cashier complains of dull ache and pressure sensation in her lower legs. It is relieved by leg elevation. She occasionally has edema in her lower legs at the end of the day. What is the most likely cause of these problems? Congestive heart failure Varicose veins Deep vein thrombosis Arterial insufficiency Which statement below is correct about pertussis? It is also called whooping cough It begins with symptoms like strep throat It lasts about 3 weeks It occurs most commonly in toddlers and young children Which of the following is the most important diagnosis to rule out in the adult patient with acute bronchitis? Pneumonia Asthma Sinusitis Pertussis A 70 year old patient presents with left lower quadrant (LLQ) abdominal pain, a markedly tender palpable abdominal wall, fever, and leukocytosis. Of the following terms, which correctly describes the suspected condition? Diverticulosis Diverticula Diverticulitis Diverticulum Sylvia, age 83, presents with a 3 day history of pain and burning in the left forehead. This morning she noticed a rash with erythematous papules in that site. What do you suspect? Varicella Herpes zoster Syphilis Rubella A 33-year-old female is admitted with acute pancreatitis. The nurse practitioner knows that the most common cause of pancreatitis is: Alcohol Gallstones Medications Pregnancy When a patient presents with symptoms of acute gallbladder disease, what is the appropriate nurse practitioner action? Order abdominal x-rays Order an abdominal ultrasound Refer the patient to a surgeon for evaluation Prescribe pain medication rder an abdominal ultrasound A false-positive result with the fecal occult blood test can result from: ingestion of large amounts of vitamin C a high dietary intake of rare cooked beef a colonic neoplasm that is not bleeding stool that has been stored before testing A 76-year-old male complains of weight loss, nausea, vomiting, abdominal cramping and pain. Physical findings include an abdominal mass and stool positive for occult blood. The nurse practitioner pain suspects a tumor in the small intestine. The best diagnostic test for this patient is: Colonoscopy Small bowel follow-through Barium enema CT abdomen A patient presents to urgent care complaining of dyspnea, fatigue, and lower extremity edema. The echocardiogram reveals and ejection fraction of 38%. The nurse practitioner knows that these findings are consistent with: Mitral regurgitation Systolic heart failure Cardiac myxoma Diastolic heart failure Maxine, Age 76, has just been given a diagnosis of pneumonia. Which of the following is an indication that she should be hospitalized? Multilobar involvement on chest x-ray with the inability to take oral medications Alert and oriented, slightly high but stable vital signs, and no one to take care of her at home Sputum and gram positive organisms A complete blood count showing leukocytosis A 55 year old man is diagnosed with basal cell carcinoma. The nurse practitioner correctly tells him: "It is the most common cause of death in patients with skin cancer." "It can be cured with surgical excision or radiation therapy." "It is a slow growing skin cancer that rarely undergoes malignant changes." "It can be cured using 5-flurouracil cream twice daily for 2 to 4 weeks." Expected spirometry readings when the patient has chronic emphysema include: Decreased residual volume (RV) Increased vital capacity (VC) Increased forced expiratory volume (FEV-1) Increased total lung capacity (TLC) An 80-year-old male admits to difficulty swallowing during the review of systems. The nurse practitioner recognizes the differential diagnosis for this patient's dysphagia is: Esophageal cancer Chest pain GERD A and C All of the above A 40 year old female with history of frequent sun exposure presents with a multicolored lesion on her back. It has irregular borders and is about 11mm in diameter. What should the nurse practitioner suspect? Squamous cell carcinoma Malignant melanoma A common nevus Basal cell carcinoma Which of the following is not a goal of treatment for the patient with cystic fibrosis? Prevent intestinal obstruction Provide adequate nutrition Promote clearance of secretions Replace water-soluble vitamins The nurse practitioner is performing a physical exam on a middle-aged African-American man. Which of the following areas is a common site for melanomas in African-Americans and other dark-skinned individuals? Scalp Nails Feet B and C All of the above An adult presents with tinea corporis. Which item below is a risk factor for its development? Topical steroid use Topical antibiotic use A recent laceration Cold climates A patient has experienced nausea and vomiting, headache, malaise, low grade fever, abdominal cramps, and watery diarrhea for 72 hours. His white count is elevated with a shift to the left. He is requesting medication for diarrhea. What is the most appropriate response? Prescribe loperamide (Immodium) or atropine-diphenoxylate (Lomotil) and a clear liquid diet for 24 hours. Prescribe a broad-spectrum antibiotic such as ciprofloxacin (Cipro), and symptom management. Offer an anti-emetic medication such as ondansetron (Zofran) and provide oral fluid and electrolyte replacement instruction. Order stool cultures. Janine, age 29, has numerous transient lesions that come and go, and she is diagnosed with urticaria. What do you order? Aspirin NSAIDs Opioids Antihistamines Of the following signs and symptoms of congestive heart failure (CHF), the earliest clinical manifestation is: Peripheral edema Weight gain Shortness of breath Nocturnal dyspnea A 16 year old male presents with mild sore throat, fever, fatigue, posterior cervical adenopathy, and palatine petechiae. Without a definitive diagnosis for this patient, what drug would be least appropriate to prescribe? Ibuprofen Erythromycin Amoxicillin Acetaminophen A 70 year old man who walks 2 miles every day complains of pain in his left calf when he is walking. The problem has gotten gradually worse and now he is unable to complete his 2 mile walk. What question asked during the history, if answered affirmatively, would suggest a diagnosis of arteriosclerosis obliterans? "Are you wearing your usual shoes?" "Do you also have chest pain when you have leg pain?" "Is your leg pain relieved by rest?" "Do you ever have the same pain in the other leg?" Which of the following statements about malignant melanomas is true? They usually occur in older adult males The patient has no family history of melanoma They are common in blacks The prognosis is directly related to the thickness of the lesion Sheila, age 78, presents with a chief complaint of waking up during the night coughing. You examine her and find an S3 heart sound, pulmonary crackles that do not clear with coughing, and peripheral edema. What do you suspect? Asthma Nocturnal allergies Valvular disease Heart failure Which antibiotic would be the most effective in treating community acquired pneumonia (CAP) in a young adult without any comorbid conditions? Erythromycin Clarithromycin (Biaxin) Doxycycline (Vibramycin) Penicillin Which of the following dermatologic vehicles are the most effective in absorbing moisture and decreasing friction? Powders Gels Creams Lotion A 70 year old patient presents with a slightly raised, scaly, erythematous patch on her forehead. She admits to having been a "sun worshiper." The nurse practitioner suspects actinic keratosis. This lesion is a precursor to: Squamous cell carcinoma Basal cell carcinoma Malignant melanoma Acne vulgaris An elderly patient is being seen in the clinic for complaint of "weak spells" relieved by sitting or lying down. How should the nurse practitioner proceed with the physical examination? Assist the patient to a standing position and take her blood pressure. Assess the patient's cranial nerves. Compare the patient's blood pressure lying first, then sitting, and then standing. Compare the amplitude of the patient's radial and pedal pulses. What oral medication might be used to treat chronic cholethiasis in a patient who is a poor candidate for surgery? Ursodiol Ibuprofen Prednisone Surgery is the only answer A 46-year-old female with a past medical history of diabetes presents with a swollen, erythematous right auricle and is diagnosed with malignant otitis externa. The nurse practitioner knows that the most likely causative organism for this patient's problem is: Staphylococcus aureus Group A beta hemolytic streptococcus Haemophilus influenza Pseudomonas aeruginosa Which of the following is not a symptom of irritable bowel syndrome? Painful diarrhea Painful constipation Cramping and abdominal pain Weight loss A patient comes in complaining of 1 week of pain in the posterior neck with difficulty turning the head to the right. What additional history is needed? Any recent trauma Difficulty swallowing Stiffness in the right shoulder Change in sleeping habits Marvin, age 56, is a smoker with diabetes. He has just been diagnosed as hypertensive. Which of the following drugs has the potential to cause the development of bronchial asthma and inhibit gluconeogenesis? ACE Inhibitor Beta Blocker Calcium channel blocker Diuretic The differential diagnosis for a patient complaining of a sore throat includes which of the following? Gonococcal infection Thrush Leukoplakia B only A, B, and C A patient presents to the primary care provider complaining of a rash on his right forehead that started yesterday and is burning and painful. The physical exam reveals an erythematous, maculopapular rash that extends over the patient's right eye to his upper right forehead. Based on the history and examination, the most likely cause of this patient's symptoms is: Rhus dermatitis Ophthalmic zoster Chemosis Optic neuritis Before initiating an HMG CoA-reductase inhibitor for hyperlipidemia, the nurse practitioner orders liver function studies. The patient's aminotransferase (ALT) is elevated. What laboratory test(s) should be ordered? Serologic markers for hepatitis Serum bilirubin Serum cholesterol with HDL and LDL A liver biopsy A patient with elevated lipids has been started on lovastatin. After 3 weeks of therapy, he calls to report generalized muscle aches. The nurse practitioner should suspect: A drug interaction Hepatic dysfunction Hypersensitivity to lovastatin Rhabdomyolysis Treatment of acute vertigo includes: Bedrest and an antihistamine Fluids and a decongestant A sedative and decongestant Rest and a low sodium diet Treatment of H.pylori includes which of the following? Proton pump inhibitor Antibiotic therapy Bismuth subsalicylate A and B A, B, and C Carl, age 78, is brought to the office by his son, who states that his father has been unable to see clearly since last night. Carl reports that his vision is "like looking through a veil." He also sees floaters and flashing lights but is not having any pain. What do you suspect? Cataracts Glaucoma Retinal detachment Iritis In order to decrease deaths from lung cancer: All smokers should be screened annually All patients should be screened annually Only high risk patients should be screened routinely Patients should be counseled to quit smoking John, age 33, has a total cholesterol level of 188 mg/dL. How often should he be screened for hypercholesterolemia? Every 5 years Every 2 years Every year Whenever blood work is done Mort is hypertensive. Which of the following factors influenced your choice of using an alpha blocker as the antihypertensive medication? Mort is black Mort also has congestive heart failure Mort has benign prostatic hyperplasia Mort has frequent migraine headaches John, age 59, presents with recurrent, sharply circumscribed red papules and plaques with a powdery white scale on the extensor aspect of his elbows and knees. What do you suspect? Actinic keratosis Eczema Psoriasis Seborrheic dermatitis Harriet, a 79-year-old woman, comes to your office every 3 months for follow up on her hypertension. Her medications include one baby aspirin daily, Lisinopril 5mg daily, and Calcium 1500 mg daily. At today's visit. Her blood pressure is 170/89. According to JNC VIII guidelines, what should you do next to control Harriet's blood pressure? Increase her Lisinopril to 20mg daily Add a thiazide diuretic to the Lisinopril 5mg daily Discontinue the Lisinopril and start a combination of ACE Inhibitor and calcium channel blocker Discontinue the Lisinopril and start a diuretic An active 65-year-old man under your care has known acquired valvular aortic stenosis and mitral regurgitation. He also has a history of infectious endocarditis. He has recently been told he needs elective replacement of his aortic valve. When he comes into the office you discover that he has 10 remaining teeth in poor repair. Your recommendation would be to: defer any further dental work until his valve replacement is completed instruct him to have dental extraction done cautiously, having no more than 2 teeth per visit removed. suggest he consult with his oral surgeon about having all the teeth removed at once and receiving appropriate antibiotic prophylaxis coordinate with his cardiac and oral surgeons to have the tooth extractions and valve replacement done at the same time to reduce the risk of anesthetic complications. Appropriate therapy for peptic ulcer disease (PUD) is: Primarily by eradication of infection Based on etiology Aimed at diminishing prostaglandin synthesis Dependent on cessation of NSAID use Shirley, age 58, has been a diabetic for 7 years. Her blood pressure is normal. Other than her diabetes medications, what would you prescribe today during her routine office visit? A calcium channel blocker A beta blocker An ACE Inhibitor No hypertension medication Medicare is a federal program administered by the Centers for Medicare and Medicaid Services (CMS). The CMS has developed guidelines for Evaluation and Management coding, which all providers are expected to follow when coding patient visits for reimbursement. Which of the following is an important consideration regarding billing practices? It is important to "undercode" so that one does not get charged with Medicare fraud The practice of "overcoding" is essential in this age of decreasing reimbursements Failing to bill for billable services will lead to unnecessarily low revenues Time spent with the patient is a very important determinant of billing A 2 year old presents with a white pupillary reflex. What is the most likely cause of this finding? Viral conjunctivitis Glaucoma Corneal abrasion Retinoblastoma Harvey has had Meniere's disease for several years. He has some hearing loss but now has persistent vertigo. What treatment might be instituted to relieve the vertigo? Pharmacological therapy A labyrinthectomy A vestibular neurectomy Wearing an earplug in the ear with the most hearing loss Which of the following is not a risk factor for coronary arterial insufficiency? Hyperhomocysteinemia Smoking Genetic factors Alcohol ingestion An 18-year-old female presents to the urgent care center complaining of severe pruritus in both eyes that started 2 days ago. Associated symptoms include a headache and fatigue. On examination, the nurse practitioner notes some clear discharge from both eyes and some erythema of the eyelids and surrounding skin. Which of the following is the most likely cause of this patient's symptoms? Allergic conjunctivitis Bacterial conjunctivitis Gonococcal conjunctivitis Viral conjunctivitis A 20 year old is diagnosed with mild persistent asthma. What drug combination would be most effective in keeping him symptom-free? A long-acting bronchodilator An inhaled corticosteroid and cromolyn Theophylline and a short acting bronchodilator A bronchodilator PRN and an inhaled corticosteroid Acute rheumatic fever is an inflammatory disease which can follow infection with: Group A Streptococcus Staphlococcus areus B-hemolytic Streptococcus Streptococcus pyogenes A 60 year old male diabetic patient presents with redness, tenderness, and edema of the left lateral aspect of his face. His left eyelid is grossly edematous. He reports history of a toothache in the past week which "is better." His temperature is 100°F and pulse is 102 bpm. The most appropriate initial action is to: Start an oral antibiotic, refer the patient to a dentist immediately, and follow up within 3 days Order mandibular x-rays and question the patient about physical abuse Start an oral antibiotic, mouth swishes with an oral anti-infective, and an analgesic Initiate a parenteral antibiotic and consider hospital admission If a patient presents with a deep aching, red eye and there is no discharge, you should suspect: Iritis Allergic conjunctivitis Viral conjunctivitis Bacterial conjunctivitis The National Cholesterol Education Program's Adult Treatment Panel III recommends that the goal for low density lipoproteins in high risk patients be less than: 160 mg/dL 130 mg/dL 100 mg/dL 70 mg/dL A patient presents with classic symptoms of gastroesophageal reflux disease (GERD). He is instructed on life style modifications and drug therapy for 8 weeks. Three months later he returns, reporting that he was "fine" as long as he took the medication. The most appropriate next step is: Referral for surgical intervention such as a partial or complete fundoplication Dependent upon how sever the practitioner believes the condition To repeat the 8 week course of drug therapy while continuing lifestyle modifications Investigation with endoscopy, manometry, and/or pH testing Group A β-hemolytic streptococcal (GABHS) pharyngitis is most common in which age group? Under 3 years of age Preschool children 6 to 12 years of age Adolescents The most appropriate treatment for a child with mild croup is: A bronchodilator An antibiotic A decongestant A cool mist vaporizer A child complains that his "throat hurts" with swallowing. His voice is very "throaty" and he is hyperextending his neck to talk. Examination reveals asymmetrical swelling of his tonsils. His uvula is deviated to the left. What is the most likely diagnosis? Peritonsillar abscess Thyroiditis Mononucleosis Epiglottitis Salmeterol (Servent) is prescribed for a patient with asthma. What is the most important teaching point about this medication? It is not effective during an acute asthma attack. It may take 2 to 3 days to begin working. This drug works within 10 minutes. This drug may be used by patients 6 years and older. Which intervention listed below is safe for long term use by an adult with constipation? Bulk-forming agents Stool softeners Laxatives Osmotic agents A 40 year old presents with a hordeolum. The nurse practitioner teaches the patient to: Apply a topical antibiotic and warm compresses. Apply cool compresses and avoid touching the hordeolum. Use an oral antibiotic and eye flushes. Apply light palpation to facilitate drainage. Sarah has allergic rhinitis and is currently being bothered by nasal congestion. Which of the following meds ordered for allergic rhinitis would be most appropriate? An antihistamine intranasal spray A decongestant nasal spray Ipratropium Omalizumab What is the Gold standard for the diagnosis of asthma? Patient's perception of clogged airways Validated quality-of-life questionnaires Bronchoscopy Spirometry A patient complains of "an aggravating cough for the past 6 weeks." There is no physiological cause for the cough. Which medication is most likely causing the cough? Methyldopa Enalapril Amlodipine Hydrochlorothiazide Stacy, age 27, states that she has painless, white, slightly raised patches in her mouth. They are probably caused by: Herpes simplex Aphthous ulcers Candidiasis Oral cancer Risk factors for acute otitis media (AOM) include all of the following except: Household cigarette smoke Group daycare attendance Sibling history of acute otitis media African-American ethnicity Which of the following can result from chronic inflammation of a meibomian gland? A chalazion Uveitis Keratitis A pterygium What conditions must be met for you to bill "incident to" the physician, receiving 100% reimbursement from Medicare? You must initiate the plan of care for the patient The physician must be on-site and engaged in patient care You must be employed as an independent contractor You must be the main health care provider who sees the patient Of the following choices, the least likely cause of cough is: Asthma Gastroesophageal reflux Acute pharyngitis Allergic rhinitis The most common correlate(s) with chronic bronchitis and emphysema is(are): Familial and genetic factors Cigarette smoking Air pollution Occupational environment Which choice below is least effective for alleviating symptoms of the common cold? Antihistamines Oral decongestants Topical decongestants Antipyretics When teaching a patient with hypertension about restricting sodium, you would include which of the following instructions? Diets with markedly reduced intakes of sodium may be associated with other beneficial effects beyond blood pressure control Sodium restriction can cause serious adverse effects A goal of 3 g of sodium chloride or 1.2 g of sodium per day is easily achievable Seventy-five of sodium intake is derived from processed foods Which of the following heart murmurs warrants the greatest concern? Systolic murmur Venous hum murmur Diastolic murmur Flow murmur A patient presents with an inflamed upper eyelid margin. The conjunctiva is red and there is particulate matter along the upper eyelid. The patient complains of a sensation that "there is something in my eye." What is the diagnosis and how should it be treated? Hordeolum; treat with a topical antibiotic and warm compress Conjunctivitis; treat with topical antibiotic and warm compresses Blepharitis; treat with warm compresses and gentle debridement with a cotton swab Chalazion; refer to an ophthalmologist for incision and drainage A 57-year-old male presents to urgent care complaining of substernal chest discomfort for the past hour. The EKG reveals ST elevations in Leads II, III, and AVF. The nurse practitioner is aware that these changes are consistent with which myocardial infarction territory? Inferior wall Anterior wall Apical wall Lateral wall The nurse practitioner observes a tympanic membrane that is opaque, has decreased mobility, and is without bulging or inflammation. The least likely diagnosis for this patient is: Acute otitis media (AOM) Otitis media with effusion Mucoid otitis media Serous otitis media Alan, age 54, notices a bulge in his midline every time he rises from bed in the morning. You tell him it is a ventral hernia, also known as: inguinal hernia epigastric hernia umbilical hernia incisional hernia A 58-year-old man is diagnosed with Barrett's esophagus after an endoscopy. He has no known allergies. Which of the following medications is MOST appropriate to treat this patient's disorder? Omeprazole Ranitidine An antacid None of the above Larry, age 66, is a smoker with hyperlipidemia and hypertension. He is 6 months post-MI. To prevent reinfarction, the most important behavior change that he can make is to: Quit smoking Maintain aggressive hypertension therapy Stick to a low-fat, low-sodium diet Continue with his exercise program Risk factors for acute arterial insufficiency include which of the following? Recent myocardial infarction Atrial fibrillation Atherosclerosis All of the above Impetigo and folliculitis are usually successfully treated with: Systemic antibiotics Topical antibiotics Topical steroid creams Cleansing and debridement [Show More]

Last updated: 1 year ago

Preview 1 out of 208 pages

Reviews( 6 )

user-profile-pic


by Esike Enjei · 2 years ago

excellent

Thank you for the review, am happy the paper assisted you. Happy study. by A+ Solutions. 2 years ago

I appreciate the review, I am happy to hit the mark for you and I'm working hard to meet your best expectations . Come back and also refer others to my page. Happy study and SUCCESS! by A+ Solutions. 1 year ago

user-profile-pic


by A+ Solutions · 2 years ago

Thank you for the review, am happy the paper assisted you. Happy study.
user-profile-pic


by klingsm · 1 year ago

Great

I appreciate the review, I am happy to hit the mark for you and I'm working hard to meet your best expectations . Come back and also refer others to my page. Happy study and SUCCESS! by A+ Solutions. 1 year ago

Much appreciated, am glad the paper was of help to you. by A+ Solutions. 1 year ago

user-profile-pic


by A+ Solutions · 1 year ago

I appreciate the review, I am happy to hit the mark for you and I'm working hard to meet your best expectations . Come back and also refer others to my page. Happy study and SUCCESS!
user-profile-pic


by A+ Solutions · 1 year ago

I appreciate the review, I am happy to hit the mark for you and I'm working hard to meet your best expectations . Come back and also refer others to my page. Happy study and SUCCESS!
user-profile-pic


by A+ Solutions · 1 year ago

Much appreciated, am glad the paper was of help to you.

$13.50

Add to cart

Instant download

Can't find what you want? Try our AI powered Search

OR

GET ASSIGNMENT HELP
346
2

Document information


Connected school, study & course


About the document


Uploaded On

Oct 19, 2021

Number of pages

208

Written in

Seller


seller-icon
A+ Solutions

Member since 3 years

164 Documents Sold


Additional information

This document has been written for:

Uploaded

Oct 19, 2021

Downloads

 2

Views

 346

Document Keyword Tags

Recommended For You


$13.50
What is Browsegrades

In Browsegrades, a student can earn by offering help to other student. Students can help other students with materials by upploading their notes and earn money.

We are here to help

We're available through e-mail, Twitter, Facebook, and live chat.
 FAQ
 Questions? Leave a message!

Follow us on
 Twitter

Copyright © Browsegrades · High quality services·